[algogeeks] Improve data store for this particular Code Chef problem to improve performance in time

2013-07-09 Thread abhishek sharma
 Given *N* integers *A1, A2, …. AN*, Dexter wants to know how many ways he
can choose three numbers such that they are three consecutive terms of an
arithmetic progression.

Meaning that, how many triplets *(i, j, k)* are there such that *1 ≤ i  j
 k ≤ N* and *Aj - Ai = Ak - Aj*.

So the triplets (2, 5, 8), (10, 8, 6), (3, 3, 3) are valid as they are
three consecutive terms of an arithmetic progression. But the triplets (2,
5, 7), (10, 6, 8) are not.
Input

First line of the input contains an integer *N (3 ≤ N ≤ 10)*. Then the
following line contains *N* space separated integers *A1, A2, …, AN* and
they have values between *1* and *3* (inclusive).
Output

Output the number of ways to choose a triplet such that they are three
consecutive terms of an arithmetic progression.
Example

*Input:*
10
3 5 3 6 3 4 10 4 5 2
*Output:*
9

*I am attaching my solution..* I reached this solution after improving
the performance 10 times.. but it is still not accepted as the time
limit exceeds 3 seconds.

Can anyone please please suggest how to improve this ...
(I do not want a different approach .. I want to improve my approach)

PS: This is a practice problem .. So I am not cheating :)


-- 
Nice Day

Abhishek Sharma
Bachelor of Technology
IIT Kanpur (2009)

-- 
You received this message because you are subscribed to the Google Groups 
Algorithm Geeks group.
To unsubscribe from this group and stop receiving emails from it, send an email 
to algogeeks+unsubscr...@googlegroups.com.




APTripletInStream.java
Description: Binary data


Re: [algogeeks] Improve data store for this particular Code Chef problem to improve performance in time

2013-07-09 Thread abhishek sharma
@Tian tanx


On Tue, Jul 9, 2013 at 5:55 PM, abhishek sharma abhishek.p...@gmail.comwrote:

 Okay let me explain my approach -

 1. Read numbers from the input stream and create an array of lists.. i
 started with hashmaps and hashsets etc .. but they greatly killed
 performance

 2. Each index in the array holds the positions of that particular index(in
 the stream) in a list..(I simply insert these positions in the list at
 array[index]..)

 3. Any such list (at any index in the array) is sorted at any given point
 of time since the positions are only going to increase as we keep reading
 input from stream..

 4. Once the input is read, iterate over this data store in the following
 manner -

 Let result holds the answer

 for i: 1 to 31 // Loop A
 if(array[i].size  2)
 result += S(S-1)(S-2)/6
 for(j: 2*i-1 to i+1)
  listRight = array[j]
  listLeft = array[i - (j-i)]
  for int x: array[i]
  //use Collections.binarySearch
  a = number of elements greater than x in listRight
  c = listRight.size - a
  b =  number of elements less than x in listLeft
  d = listLeft.Size - b
  result += a*b
  result += c*d// Explanation E

  return result once Loop A is done..

 E = Suppose 6,.,4.,2 occur in the stream as well as 2...,4...,6 ..
 I need to capture both these as they both form an AP

 To improve time, it is recommended(on CodeChef, it uses SPOJ) to use
 BufferedReader when reading long inputs from console.. Hence I am using
 that ..

 My SkypeID: abhishekpc87iit.. Feel free to contact me .. always online..


 On Tue, Jul 9, 2013 at 5:32 PM, Tian Guo tian@epfl.ch wrote:

 Can you just briefly describe your algorithm and time complexity? Then we
 could know the problem and think about from which perspective to improve it.

 Thx!


 2013/7/9 abhishek sharma abhishek.p...@gmail.com


  Given *N* integers *A1, A2, …. AN*, Dexter wants to know how many ways
 he can choose three numbers such that they are three consecutive terms of
 an arithmetic progression.

 Meaning that, how many triplets *(i, j, k)* are there such that *1 ≤ i
  j  k ≤ N* and *Aj - Ai = Ak - Aj*.

 So the triplets (2, 5, 8), (10, 8, 6), (3, 3, 3) are valid as they are
 three consecutive terms of an arithmetic progression. But the triplets (2,
 5, 7), (10, 6, 8) are not.
 Input

 First line of the input contains an integer *N (3 ≤ N ≤ 10)*. Then
 the following line contains *N* space separated integers *A1, A2, …, AN*and 
 they have values between
 *1* and *3* (inclusive).
 Output

 Output the number of ways to choose a triplet such that they are three
 consecutive terms of an arithmetic progression.
 Example

 *Input:*
 10
 3 5 3 6 3 4 10 4 5 2
 *Output:*
 9

 *I am attaching my solution..* I reached this solution after improving the 
 performance 10 times.. but it is still not accepted as the time limit 
 exceeds 3 seconds.



 Can anyone please please suggest how to improve this ...
 (I do not want a different approach .. I want to improve my approach)

 PS: This is a practice problem .. So I am not cheating :)


 --
 Nice Day

 Abhishek Sharma
 Bachelor of Technology
 IIT Kanpur (2009)

 --
 You received this message because you are subscribed to the Google
 Groups Algorithm Geeks group.
 To unsubscribe from this group and stop receiving emails from it, send
 an email to algogeeks+unsubscr...@googlegroups.com.




  --
 You received this message because you are subscribed to the Google Groups
 Algorithm Geeks group.
 To unsubscribe from this group and stop receiving emails from it, send an
 email to algogeeks+unsubscr...@googlegroups.com.






 --
 Nice Day

 Abhishek Sharma
 Bachelor of Technology
 IIT Kanpur (2009)




-- 
Nice Day

Abhishek Sharma
Bachelor of Technology
IIT Kanpur (2009)

-- 
You received this message because you are subscribed to the Google Groups 
Algorithm Geeks group.
To unsubscribe from this group and stop receiving emails from it, send an email 
to algogeeks+unsubscr...@googlegroups.com.




Re: [algogeeks] Improve data store for this particular Code Chef problem to improve performance in time

2013-07-09 Thread abhishek sharma
Okay let me explain my approach -

1. Read numbers from the input stream and create an array of lists.. i
started with hashmaps and hashsets etc .. but they greatly killed
performance

2. Each index in the array holds the positions of that particular index(in
the stream) in a list..(I simply insert these positions in the list at
array[index]..)

3. Any such list (at any index in the array) is sorted at any given point
of time since the positions are only going to increase as we keep reading
input from stream..

4. Once the input is read, iterate over this data store in the following
manner -

Let result holds the answer

for i: 1 to 31 // Loop A
if(array[i].size  2)
result += S(S-1)(S-2)/6
for(j: 2*i-1 to i+1)
 listRight = array[j]
 listLeft = array[i - (j-i)]
 for int x: array[i]
 //use Collections.binarySearch
 a = number of elements greater than x in listRight
 c = listRight.size - a
 b =  number of elements less than x in listLeft
 d = listLeft.Size - b
 result += a*b
 result += c*d// Explanation E

return result once Loop A is done..

E = Suppose 6,.,4.,2 occur in the stream as well as 2...,4...,6 ..
I need to capture both these as they both form an AP

To improve time, it is recommended(on CodeChef, it uses SPOJ) to use
BufferedReader when reading long inputs from console.. Hence I am using
that ..

My SkypeID: abhishekpc87iit.. Feel free to contact me .. always online..


On Tue, Jul 9, 2013 at 5:32 PM, Tian Guo tian@epfl.ch wrote:

 Can you just briefly describe your algorithm and time complexity? Then we
 could know the problem and think about from which perspective to improve it.

 Thx!


 2013/7/9 abhishek sharma abhishek.p...@gmail.com


  Given *N* integers *A1, A2, …. AN*, Dexter wants to know how many ways
 he can choose three numbers such that they are three consecutive terms of
 an arithmetic progression.

 Meaning that, how many triplets *(i, j, k)* are there such that *1 ≤ i 
 j  k ≤ N* and *Aj - Ai = Ak - Aj*.

 So the triplets (2, 5, 8), (10, 8, 6), (3, 3, 3) are valid as they are
 three consecutive terms of an arithmetic progression. But the triplets (2,
 5, 7), (10, 6, 8) are not.
 Input

 First line of the input contains an integer *N (3 ≤ N ≤ 10)*. Then
 the following line contains *N* space separated integers *A1, A2, …, AN*and 
 they have values between
 *1* and *3* (inclusive).
 Output

 Output the number of ways to choose a triplet such that they are three
 consecutive terms of an arithmetic progression.
 Example

 *Input:*
 10
 3 5 3 6 3 4 10 4 5 2
 *Output:*
 9

 *I am attaching my solution..* I reached this solution after improving the 
 performance 10 times.. but it is still not accepted as the time limit 
 exceeds 3 seconds.


 Can anyone please please suggest how to improve this ...
 (I do not want a different approach .. I want to improve my approach)

 PS: This is a practice problem .. So I am not cheating :)


 --
 Nice Day

 Abhishek Sharma
 Bachelor of Technology
 IIT Kanpur (2009)

 --
 You received this message because you are subscribed to the Google Groups
 Algorithm Geeks group.
 To unsubscribe from this group and stop receiving emails from it, send an
 email to algogeeks+unsubscr...@googlegroups.com.




  --
 You received this message because you are subscribed to the Google Groups
 Algorithm Geeks group.
 To unsubscribe from this group and stop receiving emails from it, send an
 email to algogeeks+unsubscr...@googlegroups.com.






-- 
Nice Day

Abhishek Sharma
Bachelor of Technology
IIT Kanpur (2009)

-- 
You received this message because you are subscribed to the Google Groups 
Algorithm Geeks group.
To unsubscribe from this group and stop receiving emails from it, send an email 
to algogeeks+unsubscr...@googlegroups.com.




Re: [algogeeks] Improve data store for this particular Code Chef problem to improve performance in time

2013-07-09 Thread abhishek sharma
Here is the link to problem - http://www.codechef.com/problems/COUNTARI

You can see my many unsuccessful attempts in All Submissions..

Ppl with successful submission have used a superb way to just use arrays to
solve this.. I hate those einsteins!


On Tue, Jul 9, 2013 at 5:56 PM, abhishek sharma abhishek.p...@gmail.comwrote:

 @Tian tanx


 On Tue, Jul 9, 2013 at 5:55 PM, abhishek sharma 
 abhishek.p...@gmail.comwrote:

 Okay let me explain my approach -

 1. Read numbers from the input stream and create an array of lists.. i
 started with hashmaps and hashsets etc .. but they greatly killed
 performance

 2. Each index in the array holds the positions of that particular
 index(in the stream) in a list..(I simply insert these positions in the
 list at array[index]..)

 3. Any such list (at any index in the array) is sorted at any given point
 of time since the positions are only going to increase as we keep reading
 input from stream..

 4. Once the input is read, iterate over this data store in the following
 manner -

 Let result holds the answer

 for i: 1 to 31 // Loop A
 if(array[i].size  2)
 result += S(S-1)(S-2)/6
 for(j: 2*i-1 to i+1)
  listRight = array[j]
  listLeft = array[i - (j-i)]
  for int x: array[i]
  //use Collections.binarySearch
  a = number of elements greater than x in listRight
  c = listRight.size - a
  b =  number of elements less than x in listLeft
  d = listLeft.Size - b
  result += a*b
  result += c*d// Explanation E

  return result once Loop A is done..

 E = Suppose 6,.,4.,2 occur in the stream as well as 2...,4...,6
 .. I need to capture both these as they both form an AP

 To improve time, it is recommended(on CodeChef, it uses SPOJ) to use
 BufferedReader when reading long inputs from console.. Hence I am using
 that ..

 My SkypeID: abhishekpc87iit.. Feel free to contact me .. always online..


 On Tue, Jul 9, 2013 at 5:32 PM, Tian Guo tian@epfl.ch wrote:

 Can you just briefly describe your algorithm and time complexity? Then
 we could know the problem and think about from which perspective to improve
 it.

 Thx!


 2013/7/9 abhishek sharma abhishek.p...@gmail.com


  Given *N* integers *A1, A2, …. AN*, Dexter wants to know how many
 ways he can choose three numbers such that they are three consecutive terms
 of an arithmetic progression.

 Meaning that, how many triplets *(i, j, k)* are there such that *1 ≤ i
  j  k ≤ N* and *Aj - Ai = Ak - Aj*.

 So the triplets (2, 5, 8), (10, 8, 6), (3, 3, 3) are valid as they are
 three consecutive terms of an arithmetic progression. But the triplets (2,
 5, 7), (10, 6, 8) are not.
 Input

 First line of the input contains an integer *N (3 ≤ N ≤ 10)*. Then
 the following line contains *N* space separated integers *A1, A2, …, AN
 * and they have values between *1* and *3* (inclusive).
 Output

 Output the number of ways to choose a triplet such that they are three
 consecutive terms of an arithmetic progression.
 Example

 *Input:*
 10
 3 5 3 6 3 4 10 4 5 2
 *Output:*
 9

 *I am attaching my solution..* I reached this solution after improving the 
 performance 10 times.. but it is still not accepted as the time limit 
 exceeds 3 seconds.




 Can anyone please please suggest how to improve this ...
 (I do not want a different approach .. I want to improve my approach)

 PS: This is a practice problem .. So I am not cheating :)


 --
 Nice Day

 Abhishek Sharma
 Bachelor of Technology
 IIT Kanpur (2009)

 --
 You received this message because you are subscribed to the Google
 Groups Algorithm Geeks group.
 To unsubscribe from this group and stop receiving emails from it, send
 an email to algogeeks+unsubscr...@googlegroups.com.




  --
 You received this message because you are subscribed to the Google
 Groups Algorithm Geeks group.
 To unsubscribe from this group and stop receiving emails from it, send
 an email to algogeeks+unsubscr...@googlegroups.com.






 --
 Nice Day

 Abhishek Sharma
 Bachelor of Technology
 IIT Kanpur (2009)




 --
 Nice Day

 Abhishek Sharma
 Bachelor of Technology
 IIT Kanpur (2009)




-- 
Nice Day

Abhishek Sharma
Bachelor of Technology
IIT Kanpur (2009)

-- 
You received this message because you are subscribed to the Google Groups 
Algorithm Geeks group.
To unsubscribe from this group and stop receiving emails from it, send an email 
to algogeeks+unsubscr...@googlegroups.com.




Re: [algogeeks] Fwd: Snapdeal placement proceedure

2012-08-29 Thread Abhishek Sharma
I appeared for snapdeal's exam on 27th.The question paper had 2
sections:aptitude and programming
In aptitiude,there were questions like
1) find time after 2pm, where two hands of clock intersect
2)find max of two numbers without using if,else or comparison operators
3)3 ants on vertices of triangle
4)delete duplicates from array
5)How many 7-digit numbers are there with digits in inc. order
6)2 aptitiude ques.
7)10 bottles with pills of weight 10g each.one bottle contains pill of
9g.find the bottle

In coding section,
1)check if a str is a rotation of other
2)all elements in array are present twice except a single element which is
present only once.find that element ?
3)one aptitude ques.
4)one question related to C output

On Tue, Aug 28, 2012 at 10:25 PM, sachin singh sachin...@gmail.com wrote:



 -- Forwarded message --
 From: sachin singh sachin...@gmail.com
 Date: Tue, Aug 28, 2012 at 10:23 PM
 Subject: Snapdeal placement proceedure
 To: algogeeks@googlegroups.com



 Can anyone tell about the recruitment process of snapdeal?
 I mean how to prepare for the written test?What kind of written test it
 would be?
 What are they asking in interview? And some pointers on what skills they
 are basically looking at when they come to hire at colleges

  --
 You received this message because you are subscribed to the Google Groups
 Algorithm Geeks group.
 To post to this group, send email to algogeeks@googlegroups.com.
 To unsubscribe from this group, send email to
 algogeeks+unsubscr...@googlegroups.com.
 For more options, visit this group at
 http://groups.google.com/group/algogeeks?hl=en.


-- 
You received this message because you are subscribed to the Google Groups 
Algorithm Geeks group.
To post to this group, send email to algogeeks@googlegroups.com.
To unsubscribe from this group, send email to 
algogeeks+unsubscr...@googlegroups.com.
For more options, visit this group at 
http://groups.google.com/group/algogeeks?hl=en.



Re: [algogeeks] Re: CISCO Written Test ??

2012-08-29 Thread Abhishek Sharma
50 questions were there in written test ( aptitude (22), c, c++, signals,
networking, 8085, 8086)
I got 35/ 50 in written test.
In interviews, they asked questions like what is polymorphism, reverse a
linked list, explain quicksort etc.

On Mon, Aug 27, 2012 at 8:40 AM, apoorv gupta apoorvcool2...@gmail.comwrote:

 Der were many  questions from electronics part also..20 apti 20 electronic
 ques 10 c/netwrking ques..So computer science people will have a tough
 luck.So revise basics of electronics. ques like half wave rectifier
 efficiency were asked


 On Sun, Aug 26, 2012 at 10:24 PM, deepikaanand swinyanand...@gmail.comwrote:

 written test will be  (apti + tech) no negative marking

 apti from time , speed and distance, probablity , mixtures , profit
 and loss (easy)
 2 qs to infer from the passage given(critical reasoning)
 1 DI set (too simple)
 and technical qs 2 simple C o/p qs
 A large %age of qs was from digital logic design , OS , networking
 only 1 qs(which layer guarantees reliable end to end transmission)
 do practice the qs given on various sites,most of the qs are just
 repeated...
 Interview process :- resume based

 --
 You received this message because you are subscribed to the Google Groups
 Algorithm Geeks group.
 To post to this group, send email to algogeeks@googlegroups.com.
 To unsubscribe from this group, send email to
 algogeeks+unsubscr...@googlegroups.com.
 For more options, visit this group at
 http://groups.google.com/group/algogeeks?hl=en.




 --
 *
 
 Thanks And Sincere Regards
 Apoorv Gupta
 Btech Final Year
 Computer Science And Engineering
 MNNIT Allahabad
 *

  --
 You received this message because you are subscribed to the Google Groups
 Algorithm Geeks group.
 To post to this group, send email to algogeeks@googlegroups.com.
 To unsubscribe from this group, send email to
 algogeeks+unsubscr...@googlegroups.com.
 For more options, visit this group at
 http://groups.google.com/group/algogeeks?hl=en.


-- 
You received this message because you are subscribed to the Google Groups 
Algorithm Geeks group.
To post to this group, send email to algogeeks@googlegroups.com.
To unsubscribe from this group, send email to 
algogeeks+unsubscr...@googlegroups.com.
For more options, visit this group at 
http://groups.google.com/group/algogeeks?hl=en.



Re: [algogeeks] Local Minima in Unsorted Array

2012-08-09 Thread Abhishek Sharma
http://courses.csail.mit.edu/6.006/spring11/lectures/lec02.pdf

On Mon, Aug 6, 2012 at 12:19 AM, dheeraj chawla dheeraj.chawla...@gmail.com
 wrote:

 hello guys, check this code n tell me if i m worng

 int localminima(int a[],int start,int end)
 {int mid;

 while(startend)
 {
   mid=(start+end)/2;
 if(a[start]a[start+1])
return(1);
 else if(a[end-1]a[end])
   return(1);
   else if(a[mid]a[mid+1])
end=mid-1;
else
start=mid+1;
 }

 if(startend)
   return 0;

 return -1;
 }

 On Mon, Aug 6, 2012 at 12:15 AM, payal gupta gpt.pa...@gmail.com wrote:

 this could help although not true for many cases as said above

 http://ideone.com/w5gjK



 On Sun, Aug 5, 2012 at 8:40 AM, Ashish Goel ashg...@gmail.com wrote:

 can you give an example of what do you mean by Local minima?
 From Dave's example, it looks like the minima of the whole array..

 Best Regards
 Ashish Goel
 Think positive and find fuel in failure
 +919985813081
 +919966006652


 On Fri, Aug 3, 2012 at 10:32 PM, shady sinv...@gmail.com wrote:

 Hi,
 Can anyone tell how to find local minima in an unsorted array ?
 Recommended solution : O(log(n))

 Shady.

 --
 You received this message because you are subscribed to the Google
 Groups Algorithm Geeks group.
 To post to this group, send email to algogeeks@googlegroups.com.
 To unsubscribe from this group, send email to
 algogeeks+unsubscr...@googlegroups.com.
 For more options, visit this group at
 http://groups.google.com/group/algogeeks?hl=en.


  --
 You received this message because you are subscribed to the Google
 Groups Algorithm Geeks group.
 To post to this group, send email to algogeeks@googlegroups.com.
 To unsubscribe from this group, send email to
 algogeeks+unsubscr...@googlegroups.com.
 For more options, visit this group at
 http://groups.google.com/group/algogeeks?hl=en.


  --
 You received this message because you are subscribed to the Google Groups
 Algorithm Geeks group.
 To post to this group, send email to algogeeks@googlegroups.com.
 To unsubscribe from this group, send email to
 algogeeks+unsubscr...@googlegroups.com.
 For more options, visit this group at
 http://groups.google.com/group/algogeeks?hl=en.


  --
 You received this message because you are subscribed to the Google Groups
 Algorithm Geeks group.
 To post to this group, send email to algogeeks@googlegroups.com.
 To unsubscribe from this group, send email to
 algogeeks+unsubscr...@googlegroups.com.
 For more options, visit this group at
 http://groups.google.com/group/algogeeks?hl=en.


-- 
You received this message because you are subscribed to the Google Groups 
Algorithm Geeks group.
To post to this group, send email to algogeeks@googlegroups.com.
To unsubscribe from this group, send email to 
algogeeks+unsubscr...@googlegroups.com.
For more options, visit this group at 
http://groups.google.com/group/algogeeks?hl=en.



Re: [algogeeks] directi paper pattern

2012-08-02 Thread Abhishek Sharma
please mail me too. Directi is coming for written test on this 8th in our
college.
Thanks in advance..

On Tue, Jul 31, 2012 at 11:48 PM, amit singh amitsingh...@gmail.com wrote:

  hi shaukat Ali ,it will be really kind if you can forward me that paper
 of directi
 my ID:amitsingh...@gmail.com


 On Tuesday, 31 July 2012 21:42:43 UTC+5:30, md shaukat ali wrote:



 On Tue, Jul 31, 2012 at 7:37 PM, deepikaanand swinyanand...@gmail.comwrote:

 can anyone tell me the pattern (selection procedure )followed by directi
 this year

 --
 You received this message because you are subscribed to the Google
 Groups Algorithm Geeks group.
 To view this discussion on the web visit https://groups.google.com/d/**
 msg/algogeeks/-/uaKshpROlGoJhttps://groups.google.com/d/msg/algogeeks/-/uaKshpROlGoJ
 .
 To post to this group, send email to algogeeks@googlegroups.com.
 To unsubscribe from this group, send email to algogeeks+unsubscribe@**
 googlegroups.com algogeeks%2bunsubscr...@googlegroups.com.
 For more options, visit this group at http://groups.google.com/**
 group/algogeeks?hl=en http://groups.google.com/group/algogeeks?hl=en.

 i have mailed u recently asked question by direct i in nit
 allahabad..make a view on it

  --
 You received this message because you are subscribed to the Google Groups
 Algorithm Geeks group.
 To view this discussion on the web visit
 https://groups.google.com/d/msg/algogeeks/-/e2nPukFWEpQJ.

 To post to this group, send email to algogeeks@googlegroups.com.
 To unsubscribe from this group, send email to
 algogeeks+unsubscr...@googlegroups.com.
 For more options, visit this group at
 http://groups.google.com/group/algogeeks?hl=en.


-- 
You received this message because you are subscribed to the Google Groups 
Algorithm Geeks group.
To post to this group, send email to algogeeks@googlegroups.com.
To unsubscribe from this group, send email to 
algogeeks+unsubscr...@googlegroups.com.
For more options, visit this group at 
http://groups.google.com/group/algogeeks?hl=en.



Re: [algogeeks] Anagram problem

2012-07-18 Thread Abhishek Sharma
sort each word in the list,then sort the whole list.
Now,sort the input word(string).
and then use binary search to find the word.

On Wed, Jul 18, 2012 at 8:59 PM, Bhupendra Dubey bhupendra@gmail.comwrote:

 Sort each of the word and form a trie , if any words comes again you get
 one sch case.


 On Wed, Jul 18, 2012 at 2:12 PM, vindhya chhabra vindhyachha...@gmail.com
  wrote:

 yes,sorry count sort will be O(n) so better.thanks

 On Wed, Jul 18, 2012 at 1:43 PM, saurabh singh saurab...@gmail.comwrote:

 ^sorting a string would be o(n^2logn) if u use q.sort.count sort would
 be better.
 Saurabh Singh
 B.Tech (Computer Science)
 MNNIT
 blog:geekinessthecoolway.blogspot.com



 On Wed, Jul 18, 2012 at 1:08 PM, vindhya chhabra 
 vindhyachha...@gmail.com wrote:

 sort the list,sort the word(use quick sort(nlogn  time))- and den
 search using binary search(logn time)
 or we can evn do by hashing-hash the word,den for every word keep
 decreasing the counter,if the hash array is zero ,anagram,else reset the
 hash array for given input for the checking the next word.


 On Wed, Jul 18, 2012 at 2:07 AM, Navin Kumar 
 algorithm.i...@gmail.comwrote:

 Assuming a preexisting list of 100 words, how would you efficiently
 see if a word received from input is an anagram of any of the 100 words?

 --
 You received this message because you are subscribed to the Google
 Groups Algorithm Geeks group.
 To view this discussion on the web visit
 https://groups.google.com/d/msg/algogeeks/-/5kuxjymYEc4J.
 To post to this group, send email to algogeeks@googlegroups.com.
 To unsubscribe from this group, send email to
 algogeeks+unsubscr...@googlegroups.com.
 For more options, visit this group at
 http://groups.google.com/group/algogeeks?hl=en.




 --
 Vindhya Chhabra




  --
 You received this message because you are subscribed to the Google
 Groups Algorithm Geeks group.
 To post to this group, send email to algogeeks@googlegroups.com.
 To unsubscribe from this group, send email to
 algogeeks+unsubscr...@googlegroups.com.
 For more options, visit this group at
 http://groups.google.com/group/algogeeks?hl=en.


  --
 You received this message because you are subscribed to the Google
 Groups Algorithm Geeks group.
 To post to this group, send email to algogeeks@googlegroups.com.
 To unsubscribe from this group, send email to
 algogeeks+unsubscr...@googlegroups.com.
 For more options, visit this group at
 http://groups.google.com/group/algogeeks?hl=en.




 --
 Vindhya Chhabra



  --
 You received this message because you are subscribed to the Google Groups
 Algorithm Geeks group.
 To post to this group, send email to algogeeks@googlegroups.com.
 To unsubscribe from this group, send email to
 algogeeks+unsubscr...@googlegroups.com.
 For more options, visit this group at
 http://groups.google.com/group/algogeeks?hl=en.




 --
 Thanks  regards
 Bhupendra



  --
 You received this message because you are subscribed to the Google Groups
 Algorithm Geeks group.
 To post to this group, send email to algogeeks@googlegroups.com.
 To unsubscribe from this group, send email to
 algogeeks+unsubscr...@googlegroups.com.
 For more options, visit this group at
 http://groups.google.com/group/algogeeks?hl=en.




-- 
Abhishek Sharma
Under-Graduate Student,
PEC University of Technology

-- 
You received this message because you are subscribed to the Google Groups 
Algorithm Geeks group.
To post to this group, send email to algogeeks@googlegroups.com.
To unsubscribe from this group, send email to 
algogeeks+unsubscr...@googlegroups.com.
For more options, visit this group at 
http://groups.google.com/group/algogeeks?hl=en.



Re: [algogeeks] Re: [Google] Finds all the elements that appear more than n/3 times

2012-07-17 Thread Abhishek Sharma
I think it can be done by using some randomized algorithm.
Divide the array into subarrays of equal size and then pick a random
element from each group.Do it 3-4 times,if random number comes out equal
for most of the times,return that element.
I haven't tried it.

On Fri, Jul 13, 2012 at 10:53 AM, Ganesh M ganesh.muniya...@gmail.comwrote:

 I guess the 
 linkhttp://valis.cs.uiuc.edu/~sariel/research/CG/applets/linear_prog/median.htmltalks
  about modified quick sort approach. Remember, if your choise of pivot
 is bad everytime, this will have a worst case performance of O(n). You
 should refer to Selection 
 Algorithmhttp://en.wikipedia.org/wiki/Selection_algorithmfor better worst 
 case performance.

 On Wednesday, July 11, 2012 3:12:07 PM UTC+8, Navin Kumar wrote:

 @sachin:

 http://valis.cs.uiuc.edu/~**sariel/research/CG/applets/**
 linear_prog/median.htmlhttp://valis.cs.uiuc.edu/~sariel/research/CG/applets/linear_prog/median.html

 On Wed, Jul 11, 2012 at 12:28 PM, sachin goyal sachingoyal@gmail.com
  wrote:

 To Mr. B
 how will you find median in O(n) time? please elaborate.


 On Wednesday, July 11, 2012 4:01:43 AM UTC+5:30, Mr.B wrote:

 I found a similar solution looking somewhere else.

 The solution for this problem is:
 a. There can be atmost 3 elements (only 3 distinct elements with each
 repeating n/3 times) -- check for this and done. -- O(n) time.
 b. There can be atmost 2 elements if not above case.

 1. Find the median of the input. O(N)
 2. Check if median element is repeated N/3 times or more -- O(n) - *{mark
 for output if yes}*
 3. partition the array based on median found above. - O(n)  --
 {partition is single step in quicksort}
 4. find median in left partition from (3). - O(n)
 5. check if median element is repeared n/3 times or more - O(n)  *{mark
 for output if yes}*
 6. find median in right partition from (3). - O(n)
 7.  check if median element is repeared n/3 times or more - O(n)  *{mark
 for output if yes}*

 its 7*O(N) = O(N) solution. Constant space.

 we need not check further down or recursively. {why? reason it.. its
 simple}


 On Wednesday, 27 June 2012 18:35:12 UTC-4, Navin Kumar wrote:


 Design an algorithm that, given a list of n elements in an array,
 finds all the elements that appear more than n/3 times in the list. The
 algorithm should run in linear time

 ( n =0 ).

 You are expected to use comparisons and achieve linear time. No
 hashing/excessive space/ and don't use standard linear time deterministic
 selection algo.

  --
 You received this message because you are subscribed to the Google
 Groups Algorithm Geeks group.
 To view this discussion on the web visit https://groups.google.com/d/**
 msg/algogeeks/-/PxIJd3So3tkJhttps://groups.google.com/d/msg/algogeeks/-/PxIJd3So3tkJ.


 To post to this group, send email to algogeeks@googlegroups.com.
 To unsubscribe from this group, send email to algogeeks+unsubscribe@**
 googlegroups.com algogeeks%2bunsubscr...@googlegroups.com.
 For more options, visit this group at http://groups.google.com/**
 group/algogeeks?hl=en http://groups.google.com/group/algogeeks?hl=en.


  --
 You received this message because you are subscribed to the Google Groups
 Algorithm Geeks group.
 To view this discussion on the web visit
 https://groups.google.com/d/msg/algogeeks/-/lZKI47459WgJ.

 To post to this group, send email to algogeeks@googlegroups.com.
 To unsubscribe from this group, send email to
 algogeeks+unsubscr...@googlegroups.com.
 For more options, visit this group at
 http://groups.google.com/group/algogeeks?hl=en.




-- 
Abhishek Sharma
Under-Graduate Student,
PEC University of Technology

-- 
You received this message because you are subscribed to the Google Groups 
Algorithm Geeks group.
To post to this group, send email to algogeeks@googlegroups.com.
To unsubscribe from this group, send email to 
algogeeks+unsubscr...@googlegroups.com.
For more options, visit this group at 
http://groups.google.com/group/algogeeks?hl=en.



Re: [algogeeks] Re: Traverse a 2-D array of strings

2012-07-15 Thread Abhishek Sharma
@atul007, but that doesn't work for extreme cases like
.sdlf.sfd.sd.f and sdf..sfdsf.sfddf

On 7/15/12, atul anand atul.87fri...@gmail.com wrote:
 1) you can count . in the input string +1 = number of tokens
 2) you can pass by reference a variable to mystrtok(string,delim,len);
 in your function at the end you can store count *len=count;
 and this len can be used in the loop.

 for(i=0;ilen;i++)

 On 7/15/12, Abhi abhi120...@gmail.com wrote:
 @atul007, number of rows represent number of tokenized strings.How do i
 know the number of tokenized strings? It depends upon input string and
 delimiter

 On Saturday, 14 July 2012 22:45:46 UTC+5:30, Abhi wrote:

 I have written a mystrtok function which takes a string and a delimiter
 as

 argument and returns an array of tokenized strings.But i don't know how
 to

 traverse that array
 Here is my code:-
 char string[50] = asdf.sdf.sdf.sdf.wer.sfd.df;
 char delim = '.';
 char **result = mystrtok( string , delim);

 for (int i=0; ??? ; i++)  //what to write in condition part
 printf(%s,result[i]);


 --
 You received this message because you are subscribed to the Google Groups
 Algorithm Geeks group.
 To view this discussion on the web visit
 https://groups.google.com/d/msg/algogeeks/-/exXJLYEOcXwJ.
 To post to this group, send email to algogeeks@googlegroups.com.
 To unsubscribe from this group, send email to
 algogeeks+unsubscr...@googlegroups.com.
 For more options, visit this group at
 http://groups.google.com/group/algogeeks?hl=en.



 --
 You received this message because you are subscribed to the Google Groups
 Algorithm Geeks group.
 To post to this group, send email to algogeeks@googlegroups.com.
 To unsubscribe from this group, send email to
 algogeeks+unsubscr...@googlegroups.com.
 For more options, visit this group at
 http://groups.google.com/group/algogeeks?hl=en.




-- 
Abhishek Sharma
Under-Graduate Student,
PEC University of Technology

-- 
You received this message because you are subscribed to the Google Groups 
Algorithm Geeks group.
To post to this group, send email to algogeeks@googlegroups.com.
To unsubscribe from this group, send email to 
algogeeks+unsubscr...@googlegroups.com.
For more options, visit this group at 
http://groups.google.com/group/algogeeks?hl=en.



Re: [algogeeks] flipkart question

2012-07-07 Thread Abhishek Sharma
@Raj: trace karke dekh na yaar when u have 3 0s and 3 6s.. the sum
distribution would look like this:

given below are the possibilities:

Combination of 1,2,3,4,5,6 with 0
1+0 = 1
2+0 = 2
3+0 = 3

OR

4+0 = 4
5+0 = 5
6+0 = 6

Combination of 1,2,3,4,5,6 with 6

1+6 = 7
2+6 = 8
3+6 = 9

OR

4+6 = 10
5+6 = 11
6+6 = 12

ho gye na evenly distribute :)

On Sat, Jul 7, 2012 at 10:24 PM, Prakhar Jain jprakha...@gmail.com wrote:

 Label 3 of the faces with 0 and other 3 faces with 6.


 --
 Prakhar Jain
 IIIT Allahabad
 B.Tech IT 3rd Year
 Mob no: +91 9454992196
 E-mail: rit2009...@iiita.ac.in
   jprakha...@gmail.com



 On Sat, Jul 7, 2012 at 12:52 AM, Hraday Sharma hradaysha...@gmail.comwrote:

 You are given 2 dice. Both are fair. One of the dice has no numbers
 printed on it. You have to label the unmarked dice such that when both the
 dice are thrown, the sum on the faces is evenly distributed between 1 and 12
  .


  --
 You received this message because you are subscribed to the Google Groups
 Algorithm Geeks group.
 To view this discussion on the web visit
 https://groups.google.com/d/msg/algogeeks/-/uXBWN7DSu_gJ.
 To post to this group, send email to algogeeks@googlegroups.com.
 To unsubscribe from this group, send email to
 algogeeks+unsubscr...@googlegroups.com.
 For more options, visit this group at
 http://groups.google.com/group/algogeeks?hl=en.


  --
 You received this message because you are subscribed to the Google Groups
 Algorithm Geeks group.
 To post to this group, send email to algogeeks@googlegroups.com.
 To unsubscribe from this group, send email to
 algogeeks+unsubscr...@googlegroups.com.
 For more options, visit this group at
 http://groups.google.com/group/algogeeks?hl=en.


-- 
You received this message because you are subscribed to the Google Groups 
Algorithm Geeks group.
To post to this group, send email to algogeeks@googlegroups.com.
To unsubscribe from this group, send email to 
algogeeks+unsubscr...@googlegroups.com.
For more options, visit this group at 
http://groups.google.com/group/algogeeks?hl=en.



Re: [algogeeks] Finding intersection of 2 linked lists

2012-07-05 Thread Abhishek Sharma
@nishant, you wrote until both the distance becomes equal.Which distances
? Could you please elaborate ?

On Thu, Jul 5, 2012 at 12:52 PM, Ashish Goel ashg...@gmail.com wrote:

 struct node* intersection( struct node *pL1, struct node* pL2)
 {
if ((!pL1) || (!pl2)) return NULL;
struct node * pL3 = NULL;
struct node* pL3Tail = NULL;
while(pL1)(pL2) {
 if (pL1-data pL2-data) pL1=pL1-next;
 else if  (pL1-data  pL2-data) pL2=pL2-next;
 else {
struct node *pNew = (struct node*)malloc(sizeof(struct node));
if !pNew return NULL; //scary
pNew-data = pL1-data; pNew-next = NULL;
if ( !pL3) pL3= pNew;
else pL3Tail-next = pNew;
pL3Tail = pNew;
}
return pL3;
 }




 }
 Best Regards
 Ashish Goel
 Think positive and find fuel in failure
 +919985813081
 +919966006652


 On Wed, Jul 4, 2012 at 10:41 PM, Abhi abhi120...@gmail.com wrote:

 Any efficient algorithm to find intersection of two linked lists.Example:
 Linked List 1)  1 - 2 - 3 - 4 - 5 - 6
 Linked List 2)  3 - 4 - 5

 Intersection 4 - 5 - 6

 --
 You received this message because you are subscribed to the Google Groups
 Algorithm Geeks group.
 To view this discussion on the web visit
 https://groups.google.com/d/msg/algogeeks/-/-8_lnGA-ZhgJ.
 To post to this group, send email to algogeeks@googlegroups.com.
 To unsubscribe from this group, send email to
 algogeeks+unsubscr...@googlegroups.com.
 For more options, visit this group at
 http://groups.google.com/group/algogeeks?hl=en.


  --
 You received this message because you are subscribed to the Google Groups
 Algorithm Geeks group.
 To post to this group, send email to algogeeks@googlegroups.com.
 To unsubscribe from this group, send email to
 algogeeks+unsubscr...@googlegroups.com.
 For more options, visit this group at
 http://groups.google.com/group/algogeeks?hl=en.




-- 
Abhishek Sharma
Under-Graduate Student,
PEC University of Technology

-- 
You received this message because you are subscribed to the Google Groups 
Algorithm Geeks group.
To post to this group, send email to algogeeks@googlegroups.com.
To unsubscribe from this group, send email to 
algogeeks+unsubscr...@googlegroups.com.
For more options, visit this group at 
http://groups.google.com/group/algogeeks?hl=en.



Re: [algogeeks] Permuatation of string in caase of duplicte string it shouldnt print duplicates permutation .

2012-07-04 Thread Abhishek Sharma
http://stackoverflow.com/questions/1900197/generating-variations-without-repetitions-permutations-in-java/


On Wed, Jul 4, 2012 at 8:16 PM, Nishant Pandey nishant.bits.me...@gmail.com
 wrote:

 the code works fine only in case of duplicates , but if we consider
 string to be non duplicates like say :abc then all permutation cant be
 obtainned .


 On Wed, Jul 4, 2012 at 12:31 PM, atul anand atul.87fri...@gmail.comwrote:

 you can use flag[256];

 now you just need to check
 loop:
 if (flag[str[i]]==0)
 {
  //swap()
  //permute function call
  //swap()
  flag[str[i]=1;
 }
 done


 On 7/4/12, atul anand atul.87fri...@gmail.com wrote:
  you can use flag[256];
 
  now you just need to check
  loop:
  flag[str[i]]==0)
  {
   //swap()
   //permute function call
   //swap()
   flag[str[i]=1;
  }
  done
 
  On 7/3/12, Nishant Pandey nishant.bits.me...@gmail.com wrote:
  1) Find all permutations of a string.
  2) Improve it so that the permutations are not repeated, Eg= string is
  
  Answer should be just  once not 4! times.
 
  i want suggestion to improve the recursive code in case of 2) case .
 
  --
  You received this message because you are subscribed to the Google
 Groups
  Algorithm Geeks group.
  To post to this group, send email to algogeeks@googlegroups.com.
  To unsubscribe from this group, send email to
  algogeeks+unsubscr...@googlegroups.com.
  For more options, visit this group at
  http://groups.google.com/group/algogeeks?hl=en.
 
 
 

 --
 You received this message because you are subscribed to the Google Groups
 Algorithm Geeks group.
 To post to this group, send email to algogeeks@googlegroups.com.
 To unsubscribe from this group, send email to
 algogeeks+unsubscr...@googlegroups.com.
 For more options, visit this group at
 http://groups.google.com/group/algogeeks?hl=en.


  --
 You received this message because you are subscribed to the Google Groups
 Algorithm Geeks group.
 To post to this group, send email to algogeeks@googlegroups.com.
 To unsubscribe from this group, send email to
 algogeeks+unsubscr...@googlegroups.com.
 For more options, visit this group at
 http://groups.google.com/group/algogeeks?hl=en.




-- 
Abhishek Sharma
Under-Graduate Student,
PEC University of Technology

-- 
You received this message because you are subscribed to the Google Groups 
Algorithm Geeks group.
To post to this group, send email to algogeeks@googlegroups.com.
To unsubscribe from this group, send email to 
algogeeks+unsubscr...@googlegroups.com.
For more options, visit this group at 
http://groups.google.com/group/algogeeks?hl=en.



Re: [algogeeks] simple FILE reading problem.

2012-07-04 Thread Abhishek Sharma
Fetch a character.
if isdigit( current_character )
  flag =1
else if current_character is any character except space

 while current_char  is not space
  current_char_position ++

On Wed, Jul 4, 2012 at 10:44 PM, Navin Kumar algorithm.i...@gmail.comwrote:

 Suppose a file.txt contains : 50 40 30 # # 5 # 10 # #

 i want to fetch only integers. How should i fetch it. I tried with fgetc
 and fscanf but it was too complicated.

 My approach: fetched one word at a time and put it into separate string
 and then i converted that string to integer(if each character of that
 string was between '0' to '9').

 Is there any simple way to do this.


 --
 You received this message because you are subscribed to the Google Groups
 Algorithm Geeks group.
 To view this discussion on the web visit
 https://groups.google.com/d/msg/algogeeks/-/btvudXnBrAIJ.
 To post to this group, send email to algogeeks@googlegroups.com.
 To unsubscribe from this group, send email to
 algogeeks+unsubscr...@googlegroups.com.
 For more options, visit this group at
 http://groups.google.com/group/algogeeks?hl=en.




-- 
Abhishek Sharma
Under-Graduate Student,
PEC University of Technology

-- 
You received this message because you are subscribed to the Google Groups 
Algorithm Geeks group.
To post to this group, send email to algogeeks@googlegroups.com.
To unsubscribe from this group, send email to 
algogeeks+unsubscr...@googlegroups.com.
For more options, visit this group at 
http://groups.google.com/group/algogeeks?hl=en.



Re: [algogeeks] simple FILE reading problem.

2012-07-04 Thread Abhishek Sharma
Please ignore the last post


Fetch a character.
if isdigit( current_character )
 add it to temp string
 flag =1
else if current_character is any character except space
 flag = 0
   while current_char  is not space
  current_char_position ++
else if current_char is space and flag = 1
   fetch last word (temp string)

On Wed, Jul 4, 2012 at 10:51 PM, Abhishek Sharma abhi120...@gmail.comwrote:

 Fetch a character.
 if isdigit( current_character )
   flag =1
 else if current_character is any character except space

  while current_char  is not space
   current_char_position ++


 On Wed, Jul 4, 2012 at 10:44 PM, Navin Kumar algorithm.i...@gmail.comwrote:

 Suppose a file.txt contains : 50 40 30 # # 5 # 10 # #

 i want to fetch only integers. How should i fetch it. I tried with fgetc
 and fscanf but it was too complicated.

 My approach: fetched one word at a time and put it into separate string
 and then i converted that string to integer(if each character of that
 string was between '0' to '9').

 Is there any simple way to do this.


 --
 You received this message because you are subscribed to the Google Groups
 Algorithm Geeks group.
 To view this discussion on the web visit
 https://groups.google.com/d/msg/algogeeks/-/btvudXnBrAIJ.
 To post to this group, send email to algogeeks@googlegroups.com.
 To unsubscribe from this group, send email to
 algogeeks+unsubscr...@googlegroups.com.
 For more options, visit this group at
 http://groups.google.com/group/algogeeks?hl=en.




 --
 Abhishek Sharma
 Under-Graduate Student,
 PEC University of Technology




-- 
Abhishek Sharma
Under-Graduate Student,
PEC University of Technology

-- 
You received this message because you are subscribed to the Google Groups 
Algorithm Geeks group.
To post to this group, send email to algogeeks@googlegroups.com.
To unsubscribe from this group, send email to 
algogeeks+unsubscr...@googlegroups.com.
For more options, visit this group at 
http://groups.google.com/group/algogeeks?hl=en.



Re: [algogeeks] Finding intersection of 2 linked lists

2012-07-04 Thread Abhishek Sharma
it was 4 - 5, not 4 - 5 - 6

On Wed, Jul 4, 2012 at 10:41 PM, Abhi abhi120...@gmail.com wrote:

 Any efficient algorithm to find intersection of two linked lists.Example:
 Linked List 1)  1 - 2 - 3 - 4 - 5 - 6
 Linked List 2)  3 - 4 - 5

 Intersection 4 - 5 - 6

 --
 You received this message because you are subscribed to the Google Groups
 Algorithm Geeks group.
 To view this discussion on the web visit
 https://groups.google.com/d/msg/algogeeks/-/-8_lnGA-ZhgJ.
 To post to this group, send email to algogeeks@googlegroups.com.
 To unsubscribe from this group, send email to
 algogeeks+unsubscr...@googlegroups.com.
 For more options, visit this group at
 http://groups.google.com/group/algogeeks?hl=en.




-- 
Abhishek Sharma
Under-Graduate Student,
PEC University of Technology

-- 
You received this message because you are subscribed to the Google Groups 
Algorithm Geeks group.
To post to this group, send email to algogeeks@googlegroups.com.
To unsubscribe from this group, send email to 
algogeeks+unsubscr...@googlegroups.com.
For more options, visit this group at 
http://groups.google.com/group/algogeeks?hl=en.



Re: [algogeeks] Finding intersection of 2 linked lists

2012-07-04 Thread Abhishek Sharma
3 - 4 - 5, sorry for that silly mistakes

On Wed, Jul 4, 2012 at 10:54 PM, Abhishek Sharma abhi120...@gmail.comwrote:

 it was 4 - 5, not 4 - 5 - 6


 On Wed, Jul 4, 2012 at 10:41 PM, Abhi abhi120...@gmail.com wrote:

 Any efficient algorithm to find intersection of two linked lists.Example:
 Linked List 1)  1 - 2 - 3 - 4 - 5 - 6
 Linked List 2)  3 - 4 - 5

 Intersection 4 - 5 - 6

 --
 You received this message because you are subscribed to the Google Groups
 Algorithm Geeks group.
 To view this discussion on the web visit
 https://groups.google.com/d/msg/algogeeks/-/-8_lnGA-ZhgJ.
 To post to this group, send email to algogeeks@googlegroups.com.
 To unsubscribe from this group, send email to
 algogeeks+unsubscr...@googlegroups.com.
 For more options, visit this group at
 http://groups.google.com/group/algogeeks?hl=en.




 --
 Abhishek Sharma
 Under-Graduate Student,
 PEC University of Technology




-- 
Abhishek Sharma
Under-Graduate Student,
PEC University of Technology

-- 
You received this message because you are subscribed to the Google Groups 
Algorithm Geeks group.
To post to this group, send email to algogeeks@googlegroups.com.
To unsubscribe from this group, send email to 
algogeeks+unsubscr...@googlegroups.com.
For more options, visit this group at 
http://groups.google.com/group/algogeeks?hl=en.



Re: [algogeeks] Re: adobe

2012-07-03 Thread Abhishek Sharma
arr1 = (int *)malloc(sizeof(int) * ncols);// memory allocated for 1st
row
arr2 = (int **)malloc(sizeof(arr1) * nrows);

I haven't tried it.So,please correct me if i am wrong

On Mon, Jul 2, 2012 at 12:55 PM, Rishabh Agarwal rishabh...@gmail.comwrote:


 nrows: number of rows
 ncols: number of columns

 int **arra = (int **)malloc( sizeof(int*) * nrows );
 int *ar = (int *)malloc( sizeof(int) * nrows * ncols );
 for( int a = 0; a  nrows; a ++ ) {
 arra[a] = ar + ncols * a;
 }

 now index of array i and j can be accessed as arra[i][j]



 On Friday, June 29, 2012 4:46:18 PM UTC+5:30, rahul r. srivastava wrote:

 implement a 2d matrix using only 2 mallocs.

  --
 You received this message because you are subscribed to the Google Groups
 Algorithm Geeks group.
 To view this discussion on the web visit
 https://groups.google.com/d/msg/algogeeks/-/Pr2cEtta_LsJ.

 To post to this group, send email to algogeeks@googlegroups.com.
 To unsubscribe from this group, send email to
 algogeeks+unsubscr...@googlegroups.com.
 For more options, visit this group at
 http://groups.google.com/group/algogeeks?hl=en.




-- 
Abhishek Sharma
Under-Graduate Student,
PEC University of Technology

-- 
You received this message because you are subscribed to the Google Groups 
Algorithm Geeks group.
To post to this group, send email to algogeeks@googlegroups.com.
To unsubscribe from this group, send email to 
algogeeks+unsubscr...@googlegroups.com.
For more options, visit this group at 
http://groups.google.com/group/algogeeks?hl=en.



Re: [algogeeks] Re: adobe

2012-07-03 Thread Abhishek Sharma
what s silly mistake. @Rahul thanks for correcting me.

On Tue, Jul 3, 2012 at 3:41 PM, rahul ranjan rahul.ranjan...@gmail.comwrote:

 @abhishek its wrong as arr1 is just a pointer o int and sizeof(arr1)
 will always be 4 bytes(size of a pointer) regardless of number of bytes
 allocated to it on heap


 On Tue, Jul 3, 2012 at 3:14 PM, Abhishek Sharma abhi120...@gmail.comwrote:

 arr1 = (int *)malloc(sizeof(int) * ncols);// memory allocated for 1st
 row
 arr2 = (int **)malloc(sizeof(arr1) * nrows);

 I haven't tried it.So,please correct me if i am wrong


 On Mon, Jul 2, 2012 at 12:55 PM, Rishabh Agarwal rishabh...@gmail.comwrote:


 nrows: number of rows
 ncols: number of columns

 int **arra = (int **)malloc( sizeof(int*) * nrows );
 int *ar = (int *)malloc( sizeof(int) * nrows * ncols );
 for( int a = 0; a  nrows; a ++ ) {
 arra[a] = ar + ncols * a;
 }

 now index of array i and j can be accessed as arra[i][j]



 On Friday, June 29, 2012 4:46:18 PM UTC+5:30, rahul r. srivastava wrote:

 implement a 2d matrix using only 2 mallocs.

  --
 You received this message because you are subscribed to the Google
 Groups Algorithm Geeks group.
 To view this discussion on the web visit
 https://groups.google.com/d/msg/algogeeks/-/Pr2cEtta_LsJ.

 To post to this group, send email to algogeeks@googlegroups.com.
 To unsubscribe from this group, send email to
 algogeeks+unsubscr...@googlegroups.com.
 For more options, visit this group at
 http://groups.google.com/group/algogeeks?hl=en.




 --
 Abhishek Sharma
 Under-Graduate Student,
 PEC University of Technology

  --
 You received this message because you are subscribed to the Google Groups
 Algorithm Geeks group.
 To post to this group, send email to algogeeks@googlegroups.com.
 To unsubscribe from this group, send email to
 algogeeks+unsubscr...@googlegroups.com.
 For more options, visit this group at
 http://groups.google.com/group/algogeeks?hl=en.


  --
 You received this message because you are subscribed to the Google Groups
 Algorithm Geeks group.
 To post to this group, send email to algogeeks@googlegroups.com.
 To unsubscribe from this group, send email to
 algogeeks+unsubscr...@googlegroups.com.
 For more options, visit this group at
 http://groups.google.com/group/algogeeks?hl=en.




-- 
Abhishek Sharma
Under-Graduate Student,
PEC University of Technology

-- 
You received this message because you are subscribed to the Google Groups 
Algorithm Geeks group.
To post to this group, send email to algogeeks@googlegroups.com.
To unsubscribe from this group, send email to 
algogeeks+unsubscr...@googlegroups.com.
For more options, visit this group at 
http://groups.google.com/group/algogeeks?hl=en.



Re: [algogeeks] Question asked in Amazon Online Test

2012-06-29 Thread Abhishek Sharma
convert prefix to infix,then convert infix to postfix.Now, to convert
prefix to infix, push numbers in one stack and operators in other.Then use
thishttp://www.velocityreviews.com/forums/t445633-prefix-to-infix-conversion.html
algo
to perform this.Then do the same for infix to postfix.It works with simple
operators,but difficult to implement with parenthesis.

On Sat, Jun 30, 2012 at 12:21 AM, rahul ranjan rahul.ranjan...@gmail.comwrote:

 oh bhai mere. kewal preorder use karke kaise tree bana dega???


 On Fri, Jun 29, 2012 at 11:23 PM, amrit harry dabbcomput...@gmail.comwrote:

 @bhaskar ur algo fails on this case (5+3)-(2+(3/6))
 -+53+2/36
 63/2+35-+
 showing that 6/3 but actually it is 3/6
 so i think it could be done by folowing algo
 make a binary tree of given expression in O(n)  then do postorder
 traversal take O(n) so problem can be solved in O(n). and take O(2*n+1)
 space

 On Fri, Jun 29, 2012 at 9:13 PM, Bhaskar Kushwaha 
 bhaskar.kushwaha2...@gmail.com wrote:

 I think just reversing the prefix notation converts it to postfix
 notation


 On Fri, Jun 29, 2012 at 7:46 PM, Gobind Kumar Hembram 
 gobind@gmail.com wrote:

 Given an integer expression in a prefix format (i.e. the operator
 precedes the number it is operating on) ,  print the expression in the
 post fix format .

 Example: If the integer expression is in the prefix format is *+56-78,
 the postfix format expression is 56+78-*. Both of these
 correspond to the expression (5+6)*(7-8).

 --
 You received this message because you are subscribed to the Google
 Groups Algorithm Geeks group.
 To post to this group, send email to algogeeks@googlegroups.com.
 To unsubscribe from this group, send email to
 algogeeks+unsubscr...@googlegroups.com.
 For more options, visit this group at
 http://groups.google.com/group/algogeeks?hl=en.




 --
 regards,
 Bhaskar Kushwaha
 Student
 Final year
 CSE
 M.N.N.I.T.  Allahabad

  --
 You received this message because you are subscribed to the Google
 Groups Algorithm Geeks group.
 To post to this group, send email to algogeeks@googlegroups.com.
 To unsubscribe from this group, send email to
 algogeeks+unsubscr...@googlegroups.com.
 For more options, visit this group at
 http://groups.google.com/group/algogeeks?hl=en.




 --
 Thanks  Regards
 Amritpal singh

  --
 You received this message because you are subscribed to the Google Groups
 Algorithm Geeks group.
 To post to this group, send email to algogeeks@googlegroups.com.
 To unsubscribe from this group, send email to
 algogeeks+unsubscr...@googlegroups.com.
 For more options, visit this group at
 http://groups.google.com/group/algogeeks?hl=en.


  --
 You received this message because you are subscribed to the Google Groups
 Algorithm Geeks group.
 To post to this group, send email to algogeeks@googlegroups.com.
 To unsubscribe from this group, send email to
 algogeeks+unsubscr...@googlegroups.com.
 For more options, visit this group at
 http://groups.google.com/group/algogeeks?hl=en.




-- 
Abhishek Sharma
Under-Graduate Student,
PEC University of Technology

-- 
You received this message because you are subscribed to the Google Groups 
Algorithm Geeks group.
To post to this group, send email to algogeeks@googlegroups.com.
To unsubscribe from this group, send email to 
algogeeks+unsubscr...@googlegroups.com.
For more options, visit this group at 
http://groups.google.com/group/algogeeks?hl=en.



Re: [algogeeks] can anyone tell me

2012-06-28 Thread Abhishek Sharma
naa noone can tell you.. haha.. just kidding...

for OS refer the prescribed text. I studied from Silberschatz, Galvin,
Gagne: *Operating System Concepts.. *
amazing book.. just understand the basics.. like process shceduling
algorithms, page shceduling algorithms, threads, context switching, virtual
memory, paging, thrashing, deadlocks etc...

On Thu, Jun 28, 2012 at 10:49 PM, Rahul verma
laptan2...@gmail.com wrote:

 can anyone tell me how to prepare OS subject for placement and interview

 which type of question interviewer will ask?

 which website will i prefer ?

 plzzz tell me ...

 --
 You received this message because you are subscribed to the Google Groups
 Algorithm Geeks group.
 To post to this group, send email to algogeeks@googlegroups.com.
 To unsubscribe from this group, send email to
 algogeeks+unsubscr...@googlegroups.com.
 For more options, visit this group at
 http://groups.google.com/group/algogeeks?hl=en.


-- 
You received this message because you are subscribed to the Google Groups 
Algorithm Geeks group.
To post to this group, send email to algogeeks@googlegroups.com.
To unsubscribe from this group, send email to 
algogeeks+unsubscr...@googlegroups.com.
For more options, visit this group at 
http://groups.google.com/group/algogeeks?hl=en.



Re: [algogeeks] Programming Question

2012-06-22 Thread Abhishek Sharma
make a hashtable where,
 key is the first character of each word and,
 value is a list which contains index of each word starting with that
character.
Now, sort that hash table wrt keys.
Now print each each key and words corresponding to that key ( given by
arr[index1], arr[index2] )

On Fri, Jun 22, 2012 at 5:55 PM, Ashot Madatyan ashot...@gmail.com wrote:

 1. read the file one char at a time, and tokenize the standalone words
 (stop
 char is either space or comma or eol)
 2. put each parsed word in a structure mapchar, vectorstring , where
 the char is the key (the first letter of each scanned word). You are
 basically creating a hash table here.
 3. now print the hash table content using the formatting of your choice.

 Rgds,
 Ashot

 -Original Message-
 From: algogeeks@googlegroups.com [mailto:algogeeks@googlegroups.com] On
 Behalf Of Gobind Kumar Hembram
 Sent: Friday, June 22, 2012 2:01 PM
 To: algogeeks@googlegroups.com
 Subject: [algogeeks] Programming Question

 I was asked this question in one company Programming Round.Please help
 me in solving this,I tried with array of pointers ,2-D array and by
 buffering.

 You have a file with names of brands separated by comma. Write a
 program (in language of your choice) to group them by their starting
 letter.

 For example, if the input file is this:
 Reebok, Puma, Adidas, Clarks, Catwalk, Converse, FILA, Lotto, Newfeel,
 Nike, Numero Uno, New Balance, ASICS, Carlton London, Crocs

 The program should print the following:
 A
  ASICS, Adidas

 C
  Carlton London, Catwalk, Clarks, Converse, Crocs

 F
  FILA

 L
  Lotto

 N
  New Balance, Newfeel, Nike, Numero Uno

 P
  Puma

 R
  Reebok

 --
 You received this message because you are subscribed to the Google Groups
 Algorithm Geeks group.
 To post to this group, send email to algogeeks@googlegroups.com.
 To unsubscribe from this group, send email to
 algogeeks+unsubscr...@googlegroups.com.
 For more options, visit this group at
 http://groups.google.com/group/algogeeks?hl=en.

 --
 You received this message because you are subscribed to the Google Groups
 Algorithm Geeks group.
 To post to this group, send email to algogeeks@googlegroups.com.
 To unsubscribe from this group, send email to
 algogeeks+unsubscr...@googlegroups.com.
 For more options, visit this group at
 http://groups.google.com/group/algogeeks?hl=en.




-- 
Abhishek Sharma
Under-Graduate Student,
PEC University of Technology

-- 
You received this message because you are subscribed to the Google Groups 
Algorithm Geeks group.
To post to this group, send email to algogeeks@googlegroups.com.
To unsubscribe from this group, send email to 
algogeeks+unsubscr...@googlegroups.com.
For more options, visit this group at 
http://groups.google.com/group/algogeeks?hl=en.



Re: [algogeeks] Re: Microsoft question

2012-06-21 Thread Abhishek Sharma
refer to this link http://www.ics.uci.edu/~eppstein/161/960130.html.
or Using quicksort , it can be done in O(n).
One more way to do this is to make min heap of size-k. Then insert elements
from the original array.If element is greater than root of the heap,swap
them.In the Last, root of the heap would be the kth smallest element

On Wed, Jun 20, 2012 at 8:47 PM, ajeet ajeet.sing...@gmail.com wrote:

 Hi,

 It looks like, The interviewer is expecting MinHeap from you,

 If modification to array is permitted just build the heap in place (from
 end bubbleUp  n-1 time) and extract K elements in sorted order
 Otherwise you need minimum O(K) memory

 Again if you want to use the quick-sort, I would prefer only using
 partition part of quick sort..
 1. Select any pivot 'P'
 2. Partition the array..
 3. position of the pivot p
 4. if p  k ( kth min lies on left part) repeat again for k
 5 if p  k ( kth min lies on right part) repeat again for k-p
 6 if p = k ( You are lucky) exit

 Again in worst case it is o(N2)

 -Ajeet

 Thanks
 -A


 On Wednesday, 20 June 2012 00:56:36 UTC+5:30, adarsh kumar wrote:

 This can be done using the concept of median of medians, wherein we can
 achieve linear time complexity in the worst case.
 This is a concept used in parallel algorithms too, check it out.

 On Mon, Jun 18, 2012 at 5:38 PM, Prem Nagarajan 
 prem.cmna...@gmail.comwrote:

 @enchantress : This problem can be solved using quicksort in O(nlogn).
 No need to go for selection sort.
 But O(n) solution is needed.


 On Mon, Jun 18, 2012 at 2:50 PM, enchantress elaenjoy...@gmail.comwrote:


 Hi all,
 A variation of selection sort can be used which takes O(nk) time.

 for i 1 to k
   min_index = i
   for j i+1 to n
  if a[j]  a[min_index]
 min_index = j
   swap(a[i],a[min_index])

 required element : a[k]

 On Sunday, 17 June 2012 08:13:18 UTC+5:30, Prem Nagarajan wrote:

 Give an array of unsorted elements, find the kth smallest element in
 the array.

 The expected time complexity is O(n) and no extra memory space should
 be used.

 Quickselect algorithm can be used to obtain the solution. Can anyone
 explain how quickselect works?

  --
 You received this message because you are subscribed to the Google
 Groups Algorithm Geeks group.
 To view this discussion on the web visit https://groups.google.com/d/**
 msg/algogeeks/-/FABBRabzVqMJhttps://groups.google.com/d/msg/algogeeks/-/FABBRabzVqMJ
 .

 To post to this group, send email to algogeeks@googlegroups.com.
 To unsubscribe from this group, send email to algogeeks+unsubscribe@**
 googlegroups.com algogeeks%2bunsubscr...@googlegroups.com.
 For more options, visit this group at http://groups.google.com/**
 group/algogeeks?hl=en http://groups.google.com/group/algogeeks?hl=en.


  --
 You received this message because you are subscribed to the Google
 Groups Algorithm Geeks group.
 To post to this group, send email to algogeeks@googlegroups.com.
 To unsubscribe from this group, send email to algogeeks+unsubscribe@**
 googlegroups.com algogeeks%2bunsubscr...@googlegroups.com.
 For more options, visit this group at http://groups.google.com/**
 group/algogeeks?hl=en http://groups.google.com/group/algogeeks?hl=en.


  --
 You received this message because you are subscribed to the Google Groups
 Algorithm Geeks group.
 To view this discussion on the web visit
 https://groups.google.com/d/msg/algogeeks/-/sBvT2ztJpoUJ.

 To post to this group, send email to algogeeks@googlegroups.com.
 To unsubscribe from this group, send email to
 algogeeks+unsubscr...@googlegroups.com.
 For more options, visit this group at
 http://groups.google.com/group/algogeeks?hl=en.




-- 
Abhishek Sharma
Under-Graduate Student,
PEC University of Technology

-- 
You received this message because you are subscribed to the Google Groups 
Algorithm Geeks group.
To post to this group, send email to algogeeks@googlegroups.com.
To unsubscribe from this group, send email to 
algogeeks+unsubscr...@googlegroups.com.
For more options, visit this group at 
http://groups.google.com/group/algogeeks?hl=en.



Re: [algogeeks] Microsoft Interview Question

2012-06-21 Thread Abhishek Sharma
find the element nearest to zero.make it pivot element.then apply one pass
of quicksort over the array.this is O(n)
On Thu, Jun 21, 2012 at 12:27 AM, Akshat Sapra sapraaks...@gmail.comwrote:

 void make_group( int a[], int size) {
   int j = 0;

  for ( int i = 0; i  size; i++ ) {
  if ( a[i]  0 ) {
 swap(a[i],a[j]);
 j++;
  }
 }
 }


 --


 Akshat Sapra
 Under Graduation(B.Tech)
 IIIT-Allahabad(Amethi Campus)
 *--*
 sapraaks...@gmail.com
 akshatsapr...@gmail.com
 rit20009008@ rit20009...@gmail.comiiita.ac.in

 --
 You received this message because you are subscribed to the Google Groups
 Algorithm Geeks group.
 To post to this group, send email to algogeeks@googlegroups.com.
 To unsubscribe from this group, send email to
 algogeeks+unsubscr...@googlegroups.com.
 For more options, visit this group at
 http://groups.google.com/group/algogeeks?hl=en.




-- 
Abhishek Sharma
Under-Graduate Student,
PEC University of Technology

-- 
You received this message because you are subscribed to the Google Groups 
Algorithm Geeks group.
To post to this group, send email to algogeeks@googlegroups.com.
To unsubscribe from this group, send email to 
algogeeks+unsubscr...@googlegroups.com.
For more options, visit this group at 
http://groups.google.com/group/algogeeks?hl=en.



Re: [algogeeks]

2012-06-21 Thread Abhishek Sharma
@umer
it's not random,but biased


On Wed, Jun 20, 2012 at 11:16 AM, Umer Farooq the.um...@gmail.com wrote:

 Hmmm, true. That's what I expected in this solution. Similarly, we can get
 3 by (3,3) (1,2)

 However, did you take a look at the other solution I proposed? I guess
 that solves the problem to some extent.


 On Tue, Jun 19, 2012 at 7:18 PM, Sourabh Singh 
 singhsourab...@gmail.comwrote:

 @Umer and Navin :
 1 is generated by (1,3) only.
 2 is generated by (1,1) and (2,3).

 so given solution is wrong


 On Tue, Jun 19, 2012 at 5:17 AM, Sourabh Singh 
 singhsourab...@gmail.comwrote:

 @ *ALL*

 please. post along with your method .
 proof than it make's equal distribution over the given range.

 On Tue, Jun 19, 2012 at 4:47 AM, Navin Kumar 
 algorithm.i...@gmail.comwrote:

 @Umer:

 rand(5) + (rand(5)%2): = it will never give 6 because for rand(7)
 range will be 0-6.
 So better try this: rand(5) + (rand(5)%3).


 On Tue, Jun 19, 2012 at 2:45 PM, Umer Farooq the.um...@gmail.comwrote:

 rand(5) + (rand(5)%2);


 On Tue, Jun 19, 2012 at 12:30 PM, Sourabh Singh 
 singhsourab...@gmail.com wrote:

 @ sry
 condition should be:

 if(20*prob = 500/7) :-)


 On Tue, Jun 19, 2012 at 12:26 AM, Sourabh Singh 
 singhsourab...@gmail.com wrote:

 @ALL

 Given a random number generator say r(5) generates number between
 1-5 uniformly at random , use it to in r(7) which should generate a 
 random
 number between 1-7 uniformly at random

 i have seen this on many site's but not a single correct solution.
 all solution's posted got rejected by someone else.:
 plz.. suggest some algo :

 my aprroach:

 let's assume a rectangle :

 100  |___
 |___|__
 500/7   |  ||
 |  ||
 |___|__|
 0 1  2  3 4  5 67
 now :

 let : num  = rand(5);
prob = rand(5);

if(prob = rand(5))
 print  num
else
 print  5 + num*(2/5)


  --
 You received this message because you are subscribed to the Google
 Groups Algorithm Geeks group.
 To post to this group, send email to algogeeks@googlegroups.com.
 To unsubscribe from this group, send email to
 algogeeks+unsubscr...@googlegroups.com.
 For more options, visit this group at
 http://groups.google.com/group/algogeeks?hl=en.


  --
 You received this message because you are subscribed to the Google
 Groups Algorithm Geeks group.
 To post to this group, send email to algogeeks@googlegroups.com.
 To unsubscribe from this group, send email to
 algogeeks+unsubscr...@googlegroups.com.
 For more options, visit this group at
 http://groups.google.com/group/algogeeks?hl=en.




 --
 Umer

  --
 You received this message because you are subscribed to the Google
 Groups Algorithm Geeks group.
 To post to this group, send email to algogeeks@googlegroups.com.
 To unsubscribe from this group, send email to
 algogeeks+unsubscr...@googlegroups.com.
 For more options, visit this group at
 http://groups.google.com/group/algogeeks?hl=en.


  --
 You received this message because you are subscribed to the Google
 Groups Algorithm Geeks group.
 To post to this group, send email to algogeeks@googlegroups.com.
 To unsubscribe from this group, send email to
 algogeeks+unsubscr...@googlegroups.com.
 For more options, visit this group at
 http://groups.google.com/group/algogeeks?hl=en.



  --
 You received this message because you are subscribed to the Google Groups
 Algorithm Geeks group.
 To post to this group, send email to algogeeks@googlegroups.com.
 To unsubscribe from this group, send email to
 algogeeks+unsubscr...@googlegroups.com.
 For more options, visit this group at
 http://groups.google.com/group/algogeeks?hl=en.




 --
 Umer

 --
 You received this message because you are subscribed to the Google Groups
 Algorithm Geeks group.
 To post to this group, send email to algogeeks@googlegroups.com.
 To unsubscribe from this group, send email to
 algogeeks+unsubscr...@googlegroups.com.
 For more options, visit this group at
 http://groups.google.com/group/algogeeks?hl=en.




-- 
Abhishek Sharma
Under-Graduate Student,
PEC University of Technology

-- 
You received this message because you are subscribed to the Google Groups 
Algorithm Geeks group.
To post to this group, send email to algogeeks@googlegroups.com.
To unsubscribe from this group, send email to 
algogeeks+unsubscr...@googlegroups.com.
For more options, visit this group at 
http://groups.google.com/group/algogeeks?hl=en.



Re: [algogeeks] Re: Reverse Queue

2012-06-20 Thread Abhishek Sharma
using recursion,

reverse(queue,front,rear) {
   if( front  rear ) {
 swap( queue[front], queue[rear] );
 reverse( queue, front+1, rear-1);
  }
}

On Wed, Jun 20, 2012 at 11:53 PM, Sreeprasad Govindankutty 
sreeprasad...@gmail.com wrote:

 Just a query :

 If the queue is implemented as an array, then is it not possible to swap
 the elements from the last and first position onwards until you reach
 middle point.  Wont this use O(1) space and O(n/2) time.



 On Wed, Jun 20, 2012 at 1:56 PM, Hassan Monfared hmonfa...@gmail.comwrote:

 void Reverse(std::queueint pQ)
 {
 if(pQ.empty())
 return;
  int item=pQ.front();
 pQ.pop();
 Reverse(pQ);
  pQ.push(item);
 }
 Regards

 On Wed, Jun 20, 2012 at 9:41 PM, enchantress elaenjoy...@gmail.comwrote:

 Queues are basically linked lists with head and tail pointers. It is
 possible to reverse the list by change of pointers in O(n) time n O(1)
 space.
 PS: Not considering queue ADT with enqueue dequeue operations.


 On Wednesday, 20 June 2012 18:34:46 UTC+5:30, Navin Kumar wrote:

 How to reverse a Queue .

 Constraints: Time complexity O(n). space complexity: O(1)

  --
 You received this message because you are subscribed to the Google
 Groups Algorithm Geeks group.
 To view this discussion on the web visit
 https://groups.google.com/d/msg/algogeeks/-/syRXPuMjBpkJ.

 To post to this group, send email to algogeeks@googlegroups.com.
 To unsubscribe from this group, send email to
 algogeeks+unsubscr...@googlegroups.com.
 For more options, visit this group at
 http://groups.google.com/group/algogeeks?hl=en.


  --
 You received this message because you are subscribed to the Google Groups
 Algorithm Geeks group.
 To post to this group, send email to algogeeks@googlegroups.com.
 To unsubscribe from this group, send email to
 algogeeks+unsubscr...@googlegroups.com.
 For more options, visit this group at
 http://groups.google.com/group/algogeeks?hl=en.


  --
 You received this message because you are subscribed to the Google Groups
 Algorithm Geeks group.
 To post to this group, send email to algogeeks@googlegroups.com.
 To unsubscribe from this group, send email to
 algogeeks+unsubscr...@googlegroups.com.
 For more options, visit this group at
 http://groups.google.com/group/algogeeks?hl=en.




-- 
Abhishek Sharma
Under-Graduate Student,
PEC University of Technology

-- 
You received this message because you are subscribed to the Google Groups 
Algorithm Geeks group.
To post to this group, send email to algogeeks@googlegroups.com.
To unsubscribe from this group, send email to 
algogeeks+unsubscr...@googlegroups.com.
For more options, visit this group at 
http://groups.google.com/group/algogeeks?hl=en.



Re: [algogeeks] Re: MS Question: Reverse stack using push, pop without any auxiliary data structure

2012-06-18 Thread Abhishek Sharma
In a stack, you can't access any element directly, except the top one.

On Mon, Jun 18, 2012 at 11:33 AM, Rituraj worstcod...@gmail.com wrote:

 My  iterative approach

 /*code in c*/
 #includestdio.h
 int main()
 {
  int stack[]={1,2,3,4,5,6,7,8},top=7;//
  int i,j,temp;

  for(i=1;i=top;i++)
  {
   temp=stack[i];

   for(j=i;j0;j--)
 stack[j]=stack[j-1];

   stack[0]=temp;
  }

  for(i=0;i=top;i++)
printf(%d ,stack[i] );

  return 0;
 }
  /*


 Rituraj
 2nd Yr.
 B.tech CSE
 NIT -Trichy

 --
 You received this message because you are subscribed to the Google Groups
 Algorithm Geeks group.
 To view this discussion on the web visit
 https://groups.google.com/d/msg/algogeeks/-/n1OE58e8B7IJ.
 To post to this group, send email to algogeeks@googlegroups.com.
 To unsubscribe from this group, send email to
 algogeeks+unsubscr...@googlegroups.com.
 For more options, visit this group at
 http://groups.google.com/group/algogeeks?hl=en.




-- 
Abhishek Sharma
Under-Graduate Student,
PEC University of Technology

-- 
You received this message because you are subscribed to the Google Groups 
Algorithm Geeks group.
To post to this group, send email to algogeeks@googlegroups.com.
To unsubscribe from this group, send email to 
algogeeks+unsubscr...@googlegroups.com.
For more options, visit this group at 
http://groups.google.com/group/algogeeks?hl=en.



Re: [algogeeks] Re: Power(n^n)

2012-06-08 Thread Abhishek Sharma
You don't need to use BigNum or long int for this program.
Both n  k should be less than 1000.
Since there is no restriction on k,you don't need Bignum
Since both n,k are restricted,you don't need bignum.
if n5, simply reject the input and return false


On Fri, Jun 8, 2012 at 11:01 AM, Dave dave_and_da...@juno.com wrote:

 @victor: But if K = 1000, then the largest N you have to deal with is 4,
 since 4^4  1000 but 5^5  1000. So your code looks like this:

 int IsNtoNEqualK( int N, int K)
 {
 return (N==1)(K==1) || (N==2)(K==4) || (N==3){K==27) ||
 (N==4)(K==256);
 }


 On Thursday, June 7, 2012 5:14:00 PM UTC-5, Victor Manuel Grijalva
 Altamirano wrote:

 Hi, everybody!!!
 I have the follow quest...

 I have two numbers N and K,  i need to check that N^N = K.
 for example:
   if N=2 and K=4 , 2^2 = 4 so return true;
   if N=3 and K=26 ,   3^3 != 26 so return false
 But 0=N , K=1000 so N^N could be have 1000 digits.

 I program in C++, and i can use Bignum (array manipulation) + fast
 power(binary power) but i want to know if exist a mathematical property.


 --
 Victor Manuel Grijalva Altamirano
 Universidad Tecnologica de La Mixteca

  --
 You received this message because you are subscribed to the Google Groups
 Algorithm Geeks group.
 To view this discussion on the web visit
 https://groups.google.com/d/msg/algogeeks/-/s6ahKx0Sxe8J.

 To post to this group, send email to algogeeks@googlegroups.com.
 To unsubscribe from this group, send email to
 algogeeks+unsubscr...@googlegroups.com.
 For more options, visit this group at
 http://groups.google.com/group/algogeeks?hl=en.




-- 
Abhishek Sharma
Under-Graduate Student,
PEC University of Technology

-- 
You received this message because you are subscribed to the Google Groups 
Algorithm Geeks group.
To post to this group, send email to algogeeks@googlegroups.com.
To unsubscribe from this group, send email to 
algogeeks+unsubscr...@googlegroups.com.
For more options, visit this group at 
http://groups.google.com/group/algogeeks?hl=en.



Re: [algogeeks] Re: Power(n^n)

2012-06-08 Thread Abhishek Sharma
Ignore the last post.
Updated:
You don't need to use BigNum or long int for this program.
Both n  k should be less than 1000.
you need bignum only if there would be no restriction on k.
Since both n,k are restricted, you don't need bignum.
if n5( 5^5  1000), simply reject the input and return false


On Fri, Jun 8, 2012 at 11:49 AM, Abhishek Sharma abhi120...@gmail.comwrote:

 You don't need to use BigNum or long int for this program.
 Both n  k should be less than 1000.
 Since there is no restriction on k,you don't need Bignum
 Since both n,k are restricted,you don't need bignum.
 if n5, simply reject the input and return false


 On Fri, Jun 8, 2012 at 11:01 AM, Dave dave_and_da...@juno.com wrote:

 @victor: But if K = 1000, then the largest N you have to deal with is 4,
 since 4^4  1000 but 5^5  1000. So your code looks like this:

 int IsNtoNEqualK( int N, int K)
 {
 return (N==1)(K==1) || (N==2)(K==4) || (N==3){K==27) ||
 (N==4)(K==256);
 }


 On Thursday, June 7, 2012 5:14:00 PM UTC-5, Victor Manuel Grijalva
 Altamirano wrote:

 Hi, everybody!!!
 I have the follow quest...

 I have two numbers N and K,  i need to check that N^N = K.
 for example:
   if N=2 and K=4 , 2^2 = 4 so return true;
   if N=3 and K=26 ,   3^3 != 26 so return false
 But 0=N , K=1000 so N^N could be have 1000 digits.

 I program in C++, and i can use Bignum (array manipulation) + fast
 power(binary power) but i want to know if exist a mathematical property.


 --
 Victor Manuel Grijalva Altamirano
 Universidad Tecnologica de La Mixteca

  --
 You received this message because you are subscribed to the Google Groups
 Algorithm Geeks group.
 To view this discussion on the web visit
 https://groups.google.com/d/msg/algogeeks/-/s6ahKx0Sxe8J.

 To post to this group, send email to algogeeks@googlegroups.com.
 To unsubscribe from this group, send email to
 algogeeks+unsubscr...@googlegroups.com.
 For more options, visit this group at
 http://groups.google.com/group/algogeeks?hl=en.




 --
 Abhishek Sharma
 Under-Graduate Student,
 PEC University of Technology




-- 
Abhishek Sharma
Under-Graduate Student,
PEC University of Technology

-- 
You received this message because you are subscribed to the Google Groups 
Algorithm Geeks group.
To post to this group, send email to algogeeks@googlegroups.com.
To unsubscribe from this group, send email to 
algogeeks+unsubscr...@googlegroups.com.
For more options, visit this group at 
http://groups.google.com/group/algogeeks?hl=en.



Re: [algogeeks] Re: What would be the output for the following code fragment?

2012-06-07 Thread Abhishek Sharma
Is there any online compiler which gives output for both little/big endian
machines ?
or it is fine to convert value from one form to another using a small c
program ?

On Thu, Jun 7, 2012 at 1:13 AM, Garima Mishra garima9...@gmail.com wrote:

 556 if the machine is little endian
 258 if machine is big endian

 On Jun 6, 11:57 pm, g4ur4v gauravyadav1...@gmail.com wrote:
  main()
  {
  int i=300;
  char *ptr = i;
  *++ptr=2;
  printf(%d,i);
 
 
 
 
 
 
 
  }

 --
 You received this message because you are subscribed to the Google Groups
 Algorithm Geeks group.
 To post to this group, send email to algogeeks@googlegroups.com.
 To unsubscribe from this group, send email to
 algogeeks+unsubscr...@googlegroups.com.
 For more options, visit this group at
 http://groups.google.com/group/algogeeks?hl=en.




-- 
Abhishek Sharma
Under-Graduate Student,
PEC University of Technology

-- 
You received this message because you are subscribed to the Google Groups 
Algorithm Geeks group.
To post to this group, send email to algogeeks@googlegroups.com.
To unsubscribe from this group, send email to 
algogeeks+unsubscr...@googlegroups.com.
For more options, visit this group at 
http://groups.google.com/group/algogeeks?hl=en.



Re: [algogeeks] Re: What would be the output for the following code fragment?

2012-06-07 Thread Abhishek Sharma
@prem, i don't get it.could you please elaborate the interesting part of
this solution ?


On Thu, Jun 7, 2012 at 11:39 AM, Abhishek Sharma abhi120...@gmail.comwrote:

 Is there any online compiler which gives output for both little/big endian
 machines ?
 or it is fine to convert value from one form to another using a small c
 program ?

 On Thu, Jun 7, 2012 at 1:13 AM, Garima Mishra garima9...@gmail.comwrote:

 556 if the machine is little endian
 258 if machine is big endian

 On Jun 6, 11:57 pm, g4ur4v gauravyadav1...@gmail.com wrote:
  main()
  {
  int i=300;
  char *ptr = i;
  *++ptr=2;
  printf(%d,i);
 
 
 
 
 
 
 
  }

 --
 You received this message because you are subscribed to the Google Groups
 Algorithm Geeks group.
 To post to this group, send email to algogeeks@googlegroups.com.
 To unsubscribe from this group, send email to
 algogeeks+unsubscr...@googlegroups.com.
 For more options, visit this group at
 http://groups.google.com/group/algogeeks?hl=en.




 --
 Abhishek Sharma
 Under-Graduate Student,
 PEC University of Technology




-- 
Abhishek Sharma
Under-Graduate Student,
PEC University of Technology

-- 
You received this message because you are subscribed to the Google Groups 
Algorithm Geeks group.
To post to this group, send email to algogeeks@googlegroups.com.
To unsubscribe from this group, send email to 
algogeeks+unsubscr...@googlegroups.com.
For more options, visit this group at 
http://groups.google.com/group/algogeeks?hl=en.



Re: [algogeeks] Re: What would be the output for the following code fragment?

2012-06-07 Thread Abhishek Sharma
oh ,now i see. 300 = 000100101100
first 8 bits = 0001
last 8 bits = 00101100

in case of big-endian machine, when we assign 2 to next location, last 8
bits become 0010 (2 in decimal), first 8 bits remain same.
in case of little-endian machine, when we assign 2 to next location, last 8
bits become 0010 (2 in decimal), last 8 bits remain same.

Am i right ?

On Thu, Jun 7, 2012 at 12:53 PM, Abhishek Sharma abhi120...@gmail.comwrote:

 @prem, i don't get it.could you please elaborate the interesting part of
 this solution ?


 On Thu, Jun 7, 2012 at 11:39 AM, Abhishek Sharma abhi120...@gmail.comwrote:

 Is there any online compiler which gives output for both little/big
 endian machines ?
 or it is fine to convert value from one form to another using a small c
 program ?

 On Thu, Jun 7, 2012 at 1:13 AM, Garima Mishra garima9...@gmail.comwrote:

 556 if the machine is little endian
 258 if machine is big endian

 On Jun 6, 11:57 pm, g4ur4v gauravyadav1...@gmail.com wrote:
  main()
  {
  int i=300;
  char *ptr = i;
  *++ptr=2;
  printf(%d,i);
 
 
 
 
 
 
 
  }

 --
 You received this message because you are subscribed to the Google
 Groups Algorithm Geeks group.
 To post to this group, send email to algogeeks@googlegroups.com.
 To unsubscribe from this group, send email to
 algogeeks+unsubscr...@googlegroups.com.
 For more options, visit this group at
 http://groups.google.com/group/algogeeks?hl=en.




 --
 Abhishek Sharma
 Under-Graduate Student,
 PEC University of Technology




 --
 Abhishek Sharma
 Under-Graduate Student,
 PEC University of Technology




-- 
Abhishek Sharma
Under-Graduate Student,
PEC University of Technology

-- 
You received this message because you are subscribed to the Google Groups 
Algorithm Geeks group.
To post to this group, send email to algogeeks@googlegroups.com.
To unsubscribe from this group, send email to 
algogeeks+unsubscr...@googlegroups.com.
For more options, visit this group at 
http://groups.google.com/group/algogeeks?hl=en.



Re: [algogeeks] If any one have algorithms for interviews by adnan aziz ebook... Please mail ...

2012-06-07 Thread Abhishek Sharma
yes,it is helpful,but read it only if u have fully understood Introduction
to algorithms or if u have strong foundation of algorithms/data structures

On Thu, Jun 7, 2012 at 12:37 PM, BUBUN SHEKHAR dce.stu...@gmail.com wrote:

 Guys is this book useful for cracking interviews??

 On Mon, Jun 4, 2012 at 1:31 AM, Dhaval Moliya moliyadha...@gmail.comwrote:

 If any one have algorithms for interviews by adnan aziz ebook... Please
 mail ...
 Thanks

 --
 You received this message because you are subscribed to the Google Groups
 Algorithm Geeks group.
 To post to this group, send email to algogeeks@googlegroups.com.
 To unsubscribe from this group, send email to
 algogeeks+unsubscr...@googlegroups.com.
 For more options, visit this group at
 http://groups.google.com/group/algogeeks?hl=en.


  --
 You received this message because you are subscribed to the Google Groups
 Algorithm Geeks group.
 To post to this group, send email to algogeeks@googlegroups.com.
 To unsubscribe from this group, send email to
 algogeeks+unsubscr...@googlegroups.com.
 For more options, visit this group at
 http://groups.google.com/group/algogeeks?hl=en.




-- 
Abhishek Sharma
Under-Graduate Student,
PEC University of Technology

-- 
You received this message because you are subscribed to the Google Groups 
Algorithm Geeks group.
To post to this group, send email to algogeeks@googlegroups.com.
To unsubscribe from this group, send email to 
algogeeks+unsubscr...@googlegroups.com.
For more options, visit this group at 
http://groups.google.com/group/algogeeks?hl=en.



[algogeeks] Abhishek Sharma wants to chat

2012-06-06 Thread Abhishek Sharma
---

Abhishek Sharma wants to stay in better touch using some of Google's coolest new
products.

If you already have Gmail or Google Talk, visit:
http://mail.google.com/mail/b-f2b6967bf6-d78b18cba1-gqPKMkil32YfIMUTyJpM7kfmucY
You'll need to click this link to be able to chat with Abhishek Sharma.

To get Gmail - a free email account from Google with over 7,500 megabytes of
storage - and chat with Abhishek Sharma, visit:
http://mail.google.com/mail/a-f2b6967bf6-d78b18cba1-gqPKMkil32YfIMUTyJpM7kfmucY?pc=en-rf---a

Gmail offers:
- Instant messaging right inside Gmail
- Powerful spam protection
- Built-in search for finding your messages and a helpful way of organizing
  emails into conversations
- No pop-up ads or untargeted banners - just text ads and related information
  that are relevant to the content of your messages

All this, and it's yours for free. But wait, there's more! By opening a Gmail
account, you also get access to Google Talk, Google's instant messaging
service:

http://www.google.com/talk/

Google Talk offers:
- Web-based chat that you can use anywhere, without a download
- A contact list that's synchronized with your Gmail account
- Free, high quality PC-to-PC voice calls when you download the Google Talk
  client

We're working hard to add new features and make improvements, so we might also
ask for your comments and suggestions periodically. We appreciate your help in
making our products even better!

Thanks,
The Google Team

To learn more about Gmail and Google Talk, visit:
http://mail.google.com/mail/help/about.html
http://www.google.com/talk/about.html

(If clicking the URLs in this message does not work, copy and paste them into
the address bar of your browser).

-- 
You received this message because you are subscribed to the Google Groups 
Algorithm Geeks group.
To post to this group, send email to algogeeks@googlegroups.com.
To unsubscribe from this group, send email to 
algogeeks+unsubscr...@googlegroups.com.
For more options, visit this group at 
http://groups.google.com/group/algogeeks?hl=en.



Re: [algogeeks] 2 Dim array as parameter

2012-06-06 Thread Abhishek Sharma
check the return value of malloc.
on success,it returns the pointer to that memory
on error, it returns NULL
..
if( (char*)malloc(10)==NULL)
{
printf(Not Enough memory available);
exit(1);
}


On Wed, Jun 6, 2012 at 7:20 PM, Ashish Goel ashg...@gmail.com wrote:

 Hi,

 traditional C style of passing 2D array to a C func is for example, void
 func(char **pArr, int m, int n).
 Like we validate a pointer before accessing it if it is valid, how do we
 verify that the array provided indeed has got memory allocated to it before
 accessing it


 Best Regards
 Ashish Goel
 Think positive and find fuel in failure
 +919985813081
 +919966006652

 --
 You received this message because you are subscribed to the Google Groups
 Algorithm Geeks group.
 To post to this group, send email to algogeeks@googlegroups.com.
 To unsubscribe from this group, send email to
 algogeeks+unsubscr...@googlegroups.com.
 For more options, visit this group at
 http://groups.google.com/group/algogeeks?hl=en.




-- 
Abhishek Sharma
Under-Graduate Student,
PEC University of Technology

-- 
You received this message because you are subscribed to the Google Groups 
Algorithm Geeks group.
To post to this group, send email to algogeeks@googlegroups.com.
To unsubscribe from this group, send email to 
algogeeks+unsubscr...@googlegroups.com.
For more options, visit this group at 
http://groups.google.com/group/algogeeks?hl=en.



Re: [algogeeks] What would be the output for the following code fragment?

2012-06-06 Thread Abhishek Sharma
http://ideone.com/Zz7ET

On Thu, Jun 7, 2012 at 12:27 AM, g4ur4v gauravyadav1...@gmail.com wrote:



 main()
 {
 int i=300;
 char *ptr = i;
 *++ptr=2;
 printf(%d,i);
 }

 --
 You received this message because you are subscribed to the Google Groups
 Algorithm Geeks group.
 To post to this group, send email to algogeeks@googlegroups.com.
 To unsubscribe from this group, send email to
 algogeeks+unsubscr...@googlegroups.com.
 For more options, visit this group at
 http://groups.google.com/group/algogeeks?hl=en.




-- 
Abhishek Sharma
Under-Graduate Student,
PEC University of Technology

-- 
You received this message because you are subscribed to the Google Groups 
Algorithm Geeks group.
To post to this group, send email to algogeeks@googlegroups.com.
To unsubscribe from this group, send email to 
algogeeks+unsubscr...@googlegroups.com.
For more options, visit this group at 
http://groups.google.com/group/algogeeks?hl=en.



[algogeeks] importance of heaps

2012-06-05 Thread Abhishek Sharma
can anyone please tell me how important are heaps as compared to other data
structures (from interview's point of view). i am not talking about simple
min/max heaps, but advanced ones like fibonacci heaps and binomial heaps

-- 
You received this message because you are subscribed to the Google Groups 
Algorithm Geeks group.
To post to this group, send email to algogeeks@googlegroups.com.
To unsubscribe from this group, send email to 
algogeeks+unsubscr...@googlegroups.com.
For more options, visit this group at 
http://groups.google.com/group/algogeeks?hl=en.



[algogeeks] implementation of dijkstra

2012-06-05 Thread Abhishek Sharma
how to implement dijkstra algorithm using fibonacci heaps ?thanks in
advance

-- 
You received this message because you are subscribed to the Google Groups 
Algorithm Geeks group.
To post to this group, send email to algogeeks@googlegroups.com.
To unsubscribe from this group, send email to 
algogeeks+unsubscr...@googlegroups.com.
For more options, visit this group at 
http://groups.google.com/group/algogeeks?hl=en.



Re: [algogeeks] If any one have algorithms for interviews by adnan aziz ebook... Please mail ...

2012-06-04 Thread Abhishek Sharma
mailing you the link for same

On Mon, Jun 4, 2012 at 1:31 AM, Dhaval Moliya moliyadha...@gmail.comwrote:

 If any one have algorithms for interviews by adnan aziz ebook... Please
 mail ...
 Thanks

 --
 You received this message because you are subscribed to the Google Groups
 Algorithm Geeks group.
 To post to this group, send email to algogeeks@googlegroups.com.
 To unsubscribe from this group, send email to
 algogeeks+unsubscr...@googlegroups.com.
 For more options, visit this group at
 http://groups.google.com/group/algogeeks?hl=en.




-- 
Abhishek Sharma
Under-Graduate Student,
PEC University of Technology

-- 
You received this message because you are subscribed to the Google Groups 
Algorithm Geeks group.
To post to this group, send email to algogeeks@googlegroups.com.
To unsubscribe from this group, send email to 
algogeeks+unsubscr...@googlegroups.com.
For more options, visit this group at 
http://groups.google.com/group/algogeeks?hl=en.



Re: [algogeeks] Re: amazon interview questions

2012-06-04 Thread Abhishek Sharma
I think it can be done by modifying the h-array and by making some changes
in KMP-algorithm

On Mon, Jun 4, 2012 at 9:35 AM, Jeevitesh jeeviteshshekha...@gmail.comwrote:

 i have not implemented it but i can you an idea how to approach it.

 Go to Each suffix in suffix or suffix array(I would prefer suffix array as
 it is easier) traverse the each suffix till you encounter the first letter
 of the suffix you are traversing and check to see this suppose i is the
 index you found out the starting letter then check
 s.substring(0,i)==s.substring(i,2i).

 I hope you get the idea.


 On Mon, Jun 4, 2012 at 9:14 AM, utsav sharma utsav.sharm...@gmail.comwrote:

 @jeevitesh :- yes i am also thinking of suffix tree,
  but i am facing problem in implementing it. did you implement it ??


 On Mon, Jun 4, 2012 at 9:11 AM, utsav sharma utsav.sharm...@gmail.comwrote:

 @hassan :- it will not work for many strings as you are checking from
 the mid of strings. try out ababcdef,aabc.
 @atul :- it should be done in O(n).


 On Sun, Jun 3, 2012 at 11:54 PM, Hassan Monfared hmonfa...@gmail.comwrote:

 yes it's not valid


 On Sun, Jun 3, 2012 at 5:36 PM, anugrah anugrah.agra...@gmail.comwrote:

 So any string with two same characters is not valid??

 for example :

 GEEK has E followed by E.

 So GEEK is also invalid?

 On Jun 3, 1:49 pm, Hassan Monfared hmonfa...@gmail.com wrote:
  bool IsValid(string s)
  {
   for(int len=0;lens.len/2;len++)
   {
 int start1=0,start2=len+1;
 while(start2s.size())
 {
for(int i=0;ilen  start2+is.size() 
  s[start1+i]=s[start2+i];i++);
if(i==len)
 return false; //not valid
start1++;
start2++;
  }
   }
  return true; // valid
 
 
 
 
 
 
 
  }
  On Sun, Jun 3, 2012 at 12:52 PM, atul anand atul.87fri...@gmail.com
 wrote:
   can be done with O(n^2) time complexity..
 
   can it be done with O(n) complexity ???
 
   On 6/3/12, utsav sharma utsav.sharm...@gmail.com wrote:
given a string tell wether it is valid or not.
string is valid if there is no substring which have the same
 substring
following it.
 
these strings are not valid:- stringstring,geek123123rt,
abcadabcad,strngstingstrngsting
 
--
You received this message because you are subscribed to the
 Google Groups
Algorithm Geeks group.
To post to this group, send email to algogeeks@googlegroups.com.
To unsubscribe from this group, send email to
algogeeks+unsubscr...@googlegroups.com.
For more options, visit this group at
   http://groups.google.com/group/algogeeks?hl=en.
 
   --
   You received this message because you are subscribed to the Google
 Groups
   Algorithm Geeks group.
   To post to this group, send email to algogeeks@googlegroups.com.
   To unsubscribe from this group, send email to
   algogeeks+unsubscr...@googlegroups.com.
   For more options, visit this group at
  http://groups.google.com/group/algogeeks?hl=en.

 --
 You received this message because you are subscribed to the Google
 Groups Algorithm Geeks group.
 To post to this group, send email to algogeeks@googlegroups.com.
 To unsubscribe from this group, send email to
 algogeeks+unsubscr...@googlegroups.com.
 For more options, visit this group at
 http://groups.google.com/group/algogeeks?hl=en.


  --
 You received this message because you are subscribed to the Google
 Groups Algorithm Geeks group.
 To post to this group, send email to algogeeks@googlegroups.com.
 To unsubscribe from this group, send email to
 algogeeks+unsubscr...@googlegroups.com.
 For more options, visit this group at
 http://groups.google.com/group/algogeeks?hl=en.



  --
 You received this message because you are subscribed to the Google Groups
 Algorithm Geeks group.
 To post to this group, send email to algogeeks@googlegroups.com.
 To unsubscribe from this group, send email to
 algogeeks+unsubscr...@googlegroups.com.
 For more options, visit this group at
 http://groups.google.com/group/algogeeks?hl=en.




 --
 *Thanks,
 Jeevitesh Shekhar Singh.*


  --
 You received this message because you are subscribed to the Google Groups
 Algorithm Geeks group.
 To post to this group, send email to algogeeks@googlegroups.com.
 To unsubscribe from this group, send email to
 algogeeks+unsubscr...@googlegroups.com.
 For more options, visit this group at
 http://groups.google.com/group/algogeeks?hl=en.




-- 
Abhishek Sharma
Under-Graduate Student,
PEC University of Technology

-- 
You received this message because you are subscribed to the Google Groups 
Algorithm Geeks group.
To post to this group, send email to algogeeks@googlegroups.com.
To unsubscribe from this group, send email to 
algogeeks+unsubscr...@googlegroups.com.
For more options, visit this group at 
http://groups.google.com/group/algogeeks?hl=en.



Re: [algogeeks] Re: find where the two list connect

2012-05-01 Thread Abhishek Sharma
@Bhupendra: your approach is correct but in case the linked lists contain
millions of nodes then this might be an overhead.

Another approach could be:

- Start with the head of of both the lists.
- Store (Hash) the addresses to which the current nodes are pointing to, in
a hashtable.
- while storing (Hashing) also check if the address already exists (for
both of them). In case it exists in the hashtable, this address (or node)
is the required node else, increment the pointers to the next nodes.

This algo will not require traversing the whole lists and will save time.

Regards,
AB

On Tue, May 1, 2012 at 9:36 PM, Umer Farooq the.um...@gmail.com wrote:

 You don't have to traverse the nodes of two lists simultaneously.

 You have to check if the every node of list one matches with the address
 of any node of list two. The first matching address will be the output.

 The worst case running time of this algo will be O(n^2)


 On Tue, May 1, 2012 at 8:47 PM, rafi rafiwie...@gmail.com wrote:

 i dont understan if i look in the pic i attached then the length of
 the first list is 5 and the length of the second list is 6.
 what should i do now?
 if i traverse the long list 5,6 nodes i dont get to the red node.
 what am i missing?

 On 1 מאי, 18:04, Bhupendra Dubey bhupendra@gmail.com wrote:
  start from head of both and  as soon as one of the list is empty means
  you
  hit null
  start counting the remaining number of nodes in the other list till that
  gets empty.
 
  Now the number obtained  above is the difference in length of the two
 list
  prior to the first common node (the red node). Now again traverse the
  longer list corresponding to the above count and then start  traversing
 the
  other list .Stop when two nodes become equal. Home!:)
 
  On Tue, May 1, 2012 at 7:55 PM, רפי וינר rafiwie...@gmail.com wrote:
   you have two linked lists that some where combine in to one list.
   pic attached to illustrate
[image: Inline image 1]
   you need to find where the two list collide. (in the pic the red node)
 
   --
   You received this message because you are subscribed to the Google
 Groups
   Algorithm Geeks group.
   To post to this group, send email to algogeeks@googlegroups.com.
   To unsubscribe from this group, send email to
   algogeeks+unsubscr...@googlegroups.com.
   For more options, visit this group at
  http://groups.google.com/group/algogeeks?hl=en.
 
  --
  bhupendra dubey
 
   Untitled.png
  14Kהצגהורדה

 --
 You received this message because you are subscribed to the Google Groups
 Algorithm Geeks group.
 To post to this group, send email to algogeeks@googlegroups.com.
 To unsubscribe from this group, send email to
 algogeeks+unsubscr...@googlegroups.com.
 For more options, visit this group at
 http://groups.google.com/group/algogeeks?hl=en.




 --
 Umer

 --
 You received this message because you are subscribed to the Google Groups
 Algorithm Geeks group.
 To post to this group, send email to algogeeks@googlegroups.com.
 To unsubscribe from this group, send email to
 algogeeks+unsubscr...@googlegroups.com.
 For more options, visit this group at
 http://groups.google.com/group/algogeeks?hl=en.


-- 
You received this message because you are subscribed to the Google Groups 
Algorithm Geeks group.
To post to this group, send email to algogeeks@googlegroups.com.
To unsubscribe from this group, send email to 
algogeeks+unsubscr...@googlegroups.com.
For more options, visit this group at 
http://groups.google.com/group/algogeeks?hl=en.



Re: [algogeeks] Re: MS QUESTION_LINKED LIST

2012-03-24 Thread Abhishek Sharma
@Atul: after u sort the list the head pointer will automatically point to
the smallest element so u actually return the head of the list.

@Sambhavna:

here is the Pseudoccode (More or less similar to, doing merge sort for
arrays):

Mersgesort(node ** list){
if( head==NULL or head- next == NULL) return;

//split the list into 2 halves (lets say *a and *b)
  split(list, a , b);

//sort the two halves individually
   mergesort(a);
   mergesort(b);

   //merge the two halves and return the smallest (first) element
   *head = sortedMerge(a,b);
}

for merging u can use recursion:
Merge(node *a, node *b){

  struct node *temp;
   if (a== NULL ) return b;
   if(b==NULL) return a;

  if(a- data = b- data)
   temp = a;
   temp- next = Merge(a-next, b);
 else
   temp = b;
   temp- next = Merge(a, b- next);
  return;
}





On Sat, Mar 24, 2012 at 1:55 PM, Sambhavna Singh coolsambha...@gmail.comwrote:

 can anyone explain vividly how we can use merge sort here. thank you.


 On Sat, Mar 24, 2012 at 1:54 PM, Sambhavna Singh 
 coolsambha...@gmail.comwrote:

 @atul: we always need to point at the next larger node..so that is ruled
 out.

 On Sat, Mar 24, 2012 at 10:14 AM, Atul Singh atulsingh7...@gmail.comwrote:

 I couldn't understand the meaning of  *return the pointer to smallest*
 
 Is it that that the pointer of largest node will point to smallest node.



 ATul Singh | Final Year  | Computer Science  Engineering | NIT
 Jalandhar  | 9530739855 |

 --
 You received this message because you are subscribed to the Google
 Groups Algorithm Geeks group.
 To post to this group, send email to algogeeks@googlegroups.com.
 To unsubscribe from this group, send email to
 algogeeks+unsubscr...@googlegroups.com.
 For more options, visit this group at
 http://groups.google.com/group/algogeeks?hl=en.



  --
 You received this message because you are subscribed to the Google Groups
 Algorithm Geeks group.
 To post to this group, send email to algogeeks@googlegroups.com.
 To unsubscribe from this group, send email to
 algogeeks+unsubscr...@googlegroups.com.
 For more options, visit this group at
 http://groups.google.com/group/algogeeks?hl=en.


-- 
You received this message because you are subscribed to the Google Groups 
Algorithm Geeks group.
To post to this group, send email to algogeeks@googlegroups.com.
To unsubscribe from this group, send email to 
algogeeks+unsubscr...@googlegroups.com.
For more options, visit this group at 
http://groups.google.com/group/algogeeks?hl=en.



Re: [algogeeks] MS QUESTION_LINKED LIST

2012-03-23 Thread Abhishek Sharma
It is basically sorting the linked list. Do not change the first pointer of
nodes and use the second pointer for sorting. return the pointer to the
smallest element. That's it.

On Sat, Mar 24, 2012 at 12:50 AM, Atul Singh atulsingh7...@gmail.comwrote:

 Given a linked list with each node having two pointers : one pointing to
 next node  other to null;
 how will u point the second pointer to next larger no. and return the
 pointer to smallest node



 --
 ATul Singh | Final Year  | Computer Science  Engineering | NIT Jalandhar
  | 9530739855 |

 --
 You received this message because you are subscribed to the Google Groups
 Algorithm Geeks group.
 To post to this group, send email to algogeeks@googlegroups.com.
 To unsubscribe from this group, send email to
 algogeeks+unsubscr...@googlegroups.com.
 For more options, visit this group at
 http://groups.google.com/group/algogeeks?hl=en.


-- 
You received this message because you are subscribed to the Google Groups 
Algorithm Geeks group.
To post to this group, send email to algogeeks@googlegroups.com.
To unsubscribe from this group, send email to 
algogeeks+unsubscr...@googlegroups.com.
For more options, visit this group at 
http://groups.google.com/group/algogeeks?hl=en.



Re: [algogeeks] Re: MS QUESTION_LINKED LIST

2012-03-23 Thread Abhishek Sharma
@don: inplace Mergesort can be used. Complexity would be O(nlogn).
@Ashish: Heapsort is reliable but unstable and also, slower.

On Sat, Mar 24, 2012 at 1:49 AM, Don dondod...@gmail.com wrote:

 A merge sort will be O(n*log n) and not use the extra memory required
 for a heap.
 Don

 On Mar 23, 3:11 pm, Ashish Goel ashg...@gmail.com wrote:
  actually, multimap can be avoided, each element of heap is key,value
 where
  key is the element and value is address and build heap on key.
  Best Regards
  Ashish Goel
  Think positive and find fuel in failure
  +919985813081
  +919966006652
 
 
 
  On Sat, Mar 24, 2012 at 1:40 AM, Ashish Goel ashg...@gmail.com wrote:
   don't know if i am complicating..assumption,
 
   build a multimap of values and the corresponding node address as well
 as a
   heap from the given nodes in first pass.
 
   now from minheap pick one by one and set the second pointer of previous
   picked min element to this element using multimap(remove from multimap
 in
   parallel while updating the second pointers).
 
   Best Regards
   Ashish Goel
   Think positive and find fuel in failure
   +919985813081
   +919966006652
 
   On Sat, Mar 24, 2012 at 12:50 AM, Atul Singh atulsingh7...@gmail.com
 wrote:
 
   Given a linked list with each node having two pointers : one pointing
 to
   next node  other to null;
   how will u point the second pointer to next larger no. and return the
   pointer to smallest node
 
   --
   ATul Singh | Final Year  | Computer Science  Engineering | NIT
   Jalandhar  | 9530739855 |
 
   --
   You received this message because you are subscribed to the Google
 Groups
   Algorithm Geeks group.
   To post to this group, send email to algogeeks@googlegroups.com.
   To unsubscribe from this group, send email to
   algogeeks+unsubscr...@googlegroups.com.
   For more options, visit this group at
  http://groups.google.com/group/algogeeks?hl=en.- Hide quoted text -
 
  - Show quoted text -

 --
 You received this message because you are subscribed to the Google Groups
 Algorithm Geeks group.
 To post to this group, send email to algogeeks@googlegroups.com.
 To unsubscribe from this group, send email to
 algogeeks+unsubscr...@googlegroups.com.
 For more options, visit this group at
 http://groups.google.com/group/algogeeks?hl=en.



-- 
You received this message because you are subscribed to the Google Groups 
Algorithm Geeks group.
To post to this group, send email to algogeeks@googlegroups.com.
To unsubscribe from this group, send email to 
algogeeks+unsubscr...@googlegroups.com.
For more options, visit this group at 
http://groups.google.com/group/algogeeks?hl=en.



Re: [algogeeks] subset of an array

2011-10-10 Thread abhishek sharma
yea DP will be O(given no * n) if all array entries are positive and the
given no is non negative.

On Mon, Oct 10, 2011 at 11:09 PM, anshu mishra anshumishra6...@gmail.comwrote:

 the simplest code could be for this question is

 void printAllSubsetSum(int ar[], int n, int x)
 {
 for (i = 0; i  (1n); i++)
 {
 int sum = 0;
  for (j = 0; j  n; j++)
  {
 if ( (1  j)  i) sum += ar[j];
  }
  if (sum == x)
  {
 for (j = 0; j  n; j++)
{
if ( (1  j)  i) printf(%d , ar[j]);
}
   printf(\n);
  }
 }
 }

 Time complexity O(2^n)

 this can be solved using DP in O(n * given number);

  --
 You received this message because you are subscribed to the Google Groups
 Algorithm Geeks group.
 To post to this group, send email to algogeeks@googlegroups.com.
 To unsubscribe from this group, send email to
 algogeeks+unsubscr...@googlegroups.com.
 For more options, visit this group at
 http://groups.google.com/group/algogeeks?hl=en.




-- 
Nice Day

Abhishek Sharma
Bachelor of Technology
IIT Kanpur (2009)

-- 
You received this message because you are subscribed to the Google Groups 
Algorithm Geeks group.
To post to this group, send email to algogeeks@googlegroups.com.
To unsubscribe from this group, send email to 
algogeeks+unsubscr...@googlegroups.com.
For more options, visit this group at 
http://groups.google.com/group/algogeeks?hl=en.



[algogeeks] Given a String with unnecessary spaces, compact it in place

2011-10-10 Thread abhishek sharma
Can in place compaction be done without left shifts?



-- 
Nice Day

Abhishek Sharma
Bachelor of Technology
IIT Kanpur (2009)

-- 
You received this message because you are subscribed to the Google Groups 
Algorithm Geeks group.
To post to this group, send email to algogeeks@googlegroups.com.
To unsubscribe from this group, send email to 
algogeeks+unsubscr...@googlegroups.com.
For more options, visit this group at 
http://groups.google.com/group/algogeeks?hl=en.



Re: [algogeeks] Re: All valid dictionary words must be found and printed.

2011-10-05 Thread abhishek sharma
if we use brute force we have sum(n + n-1 + .. n-r .. + 1) = n*n words which
are to be checked. Therefore O(n-sq).

now, if i can use a dictionary interface to reject some prefix altogether,
than i need not check some words, o/w with the given interface we cannot do
it any better than quadratic time.

On Tue, Oct 4, 2011 at 6:23 PM, Navneet navneetn...@gmail.com wrote:

 What is the source of this question?

 On Sep 20, 4:49 am, Ankur Garg ankurga...@gmail.com wrote:
  nice find bhanu..though i didnt get much :P on first read :D :D
 
  On Tue, Sep 20, 2011 at 4:34 AM, Bhanu Kishore bhanukishor...@gmail.com
 wrote:
 
 
 
 
 
 
 
   See this algorithm:
  http://en.wikipedia.org/wiki/Aho%E2%80%93Corasick_string_matching_alg.
 ..
 
   --
   You received this message because you are subscribed to the Google
 Groups
   Algorithm Geeks group.
   To post to this group, send email to algogeeks@googlegroups.com.
   To unsubscribe from this group, send email to
   algogeeks+unsubscr...@googlegroups.com.
   For more options, visit this group at
  http://groups.google.com/group/algogeeks?hl=en.

 --
 You received this message because you are subscribed to the Google Groups
 Algorithm Geeks group.
 To post to this group, send email to algogeeks@googlegroups.com.
 To unsubscribe from this group, send email to
 algogeeks+unsubscr...@googlegroups.com.
 For more options, visit this group at
 http://groups.google.com/group/algogeeks?hl=en.




-- 
Nice Day

Abhishek Sharma
Bachelor of Technology
IIT Kanpur (2009)

-- 
You received this message because you are subscribed to the Google Groups 
Algorithm Geeks group.
To post to this group, send email to algogeeks@googlegroups.com.
To unsubscribe from this group, send email to 
algogeeks+unsubscr...@googlegroups.com.
For more options, visit this group at 
http://groups.google.com/group/algogeeks?hl=en.



Re: [algogeeks] Re: Sudoku

2011-10-04 Thread abhishek sharma
Hi Don,

How is your method better than backtracking?

i hope you are implementing a sudoku solver..

Let me know.

Regds.

On Tue, Oct 4, 2011 at 10:42 PM, Don dondod...@gmail.com wrote:

 When you say Simulate Sudoku, do you mean solve a given Sudoku
 problem?

 Here is an overview of how I did that:

 I used an array of 81 integers to represent the board.
 Then I built a 27x9 table of all the groups: 9 rows, 9 columns, and 9
 squares.
 Then I built a 81x3 map which relates each location on the board to
 the 3 groups it belongs to.
 I maintain an array of 27 integers called avail whose bits indicate
 which values are still needed in that group.

 Read in the given values and update avail accordingly.

 Then repeatedly do the following until the problem is solved
   For each empty cell
   Compute the bitwise AND of the avail values for the 3 groups it
 belongs to.
   If the AND is zero, no value can go there. Return failure.
   If exactly one value can go there, put it there and update the
 avail values for the 3 groups
   If the loop above did not fill in any cells, then do the following
   Loop at each of the 27 groups
   For each value missing in that group, count the locations
 where it could go
   If it could go in exactly one location, put it there
   If it cannot go in any location, return failure.
   If the neither method above filled in any cells, then do the
 following:
   Pick the empty cell with the fewest possible values
   Try the possible values in that cell until you find one which
 allows the puzzle to be completed

 If the puzzle is solvable, this will solve it in a fraction of a
 second.

 Don

 On Oct 4, 9:21 am, himanshu kansal himanshukansal...@gmail.com
 wrote:
  can anybody give me the steps you need to check while writing a
  program to simulate sudoku
 
  i don't want the exact codejust algorithm would me more than
  sufficient.
 
  suggest also the suitable languages for implementing that..VB or
  java or any other

 --
 You received this message because you are subscribed to the Google Groups
 Algorithm Geeks group.
 To post to this group, send email to algogeeks@googlegroups.com.
 To unsubscribe from this group, send email to
 algogeeks+unsubscr...@googlegroups.com.
 For more options, visit this group at
 http://groups.google.com/group/algogeeks?hl=en.




-- 
Nice Day

Abhishek Sharma
Bachelor of Technology
IIT Kanpur (2009)

-- 
You received this message because you are subscribed to the Google Groups 
Algorithm Geeks group.
To post to this group, send email to algogeeks@googlegroups.com.
To unsubscribe from this group, send email to 
algogeeks+unsubscr...@googlegroups.com.
For more options, visit this group at 
http://groups.google.com/group/algogeeks?hl=en.



Re: [algogeeks] binary tree

2011-09-18 Thread abhishek sharma
hi prasanth,

i was asked a similar ques but with the condition that path shud terminate
at a leaf node  u just have to return true if you are able to find a path.

what i did was,
- do an inorder traversal (u have to pass a parameter that represents
sum as well along with treenode pointer)
- at every leaf check the value of cumulative sum of all the parents'
data and the current node's data

for printing paths u can use a stack



On Sun, Sep 18, 2011 at 2:26 PM, prasanth n nprasnt...@gmail.com wrote:

 You are given a binary tree in which each node contains a value. Design an
 ALGORITHM to print all paths which sum up to that value. Note that it can be
 any path in the tree - it does not have to start at the root.

 --
 *prasanth*

 --
 You received this message because you are subscribed to the Google Groups
 Algorithm Geeks group.
 To post to this group, send email to algogeeks@googlegroups.com.
 To unsubscribe from this group, send email to
 algogeeks+unsubscr...@googlegroups.com.
 For more options, visit this group at
 http://groups.google.com/group/algogeeks?hl=en.




-- 
Nice Day

Abhishek Sharma
Bachelor of Technology
IIT Kanpur (2009)

-- 
You received this message because you are subscribed to the Google Groups 
Algorithm Geeks group.
To post to this group, send email to algogeeks@googlegroups.com.
To unsubscribe from this group, send email to 
algogeeks+unsubscr...@googlegroups.com.
For more options, visit this group at 
http://groups.google.com/group/algogeeks?hl=en.



Re: [algogeeks] binary tree

2011-09-18 Thread abhishek sharma
for your question -


On Sun, Sep 18, 2011 at 3:44 PM, abhishek sharma abhishek.p...@gmail.comwrote:

 hi prasanth,

 i was asked a similar ques but with the condition that path shud terminate
 at a leaf node  u just have to return true if you are able to find a path.

 what i did was,
 - do an inorder traversal (u have to pass a parameter that represents
 sum as well along with treenode pointer)
 - at every leaf check the value of cumulative sum of all the parents'
 data and the current node's data

 for printing paths u can use a stack




 On Sun, Sep 18, 2011 at 2:26 PM, prasanth n nprasnt...@gmail.com wrote:

 You are given a binary tree in which each node contains a value. Design an
 ALGORITHM to print all paths which sum up to that value. Note that it can be
 any path in the tree - it does not have to start at the root.

 --
 *prasanth*

 --
 You received this message because you are subscribed to the Google Groups
 Algorithm Geeks group.
 To post to this group, send email to algogeeks@googlegroups.com.
 To unsubscribe from this group, send email to
 algogeeks+unsubscr...@googlegroups.com.
 For more options, visit this group at
 http://groups.google.com/group/algogeeks?hl=en.




 --
 Nice Day

 Abhishek Sharma
 Bachelor of Technology
 IIT Kanpur (2009)




-- 
Nice Day

Abhishek Sharma
Bachelor of Technology
IIT Kanpur (2009)

-- 
You received this message because you are subscribed to the Google Groups 
Algorithm Geeks group.
To post to this group, send email to algogeeks@googlegroups.com.
To unsubscribe from this group, send email to 
algogeeks+unsubscr...@googlegroups.com.
For more options, visit this group at 
http://groups.google.com/group/algogeeks?hl=en.



Re: [algogeeks] Elitmus test Help

2011-08-24 Thread Abhishek Sharma
similar to CAT though the level is little low... no technical questions are
asked..

On Wed, Aug 24, 2011 at 11:50 PM, Akanksha . akanksha...@gmail.com wrote:

 It is  a general aptitude test.. they ask u ques on quant, verbel n
 problems solving skills.. prepare well if u r planning to take this
 test as ques r tough..

 On Wed, Aug 24, 2011 at 11:45 PM, rohit rajuljain...@gmail.com wrote:
  Is anybody have any idea about pattern of elitmus test , Is It a C
  programming test or General aptitude test?
 
  --
  You received this message because you are subscribed to the Google Groups
 Algorithm Geeks group.
  To post to this group, send email to algogeeks@googlegroups.com.
  To unsubscribe from this group, send email to
 algogeeks+unsubscr...@googlegroups.com.
  For more options, visit this group at
 http://groups.google.com/group/algogeeks?hl=en.
 
 

 --
 You received this message because you are subscribed to the Google Groups
 Algorithm Geeks group.
 To post to this group, send email to algogeeks@googlegroups.com.
 To unsubscribe from this group, send email to
 algogeeks+unsubscr...@googlegroups.com.
 For more options, visit this group at
 http://groups.google.com/group/algogeeks?hl=en.



-- 
You received this message because you are subscribed to the Google Groups 
Algorithm Geeks group.
To post to this group, send email to algogeeks@googlegroups.com.
To unsubscribe from this group, send email to 
algogeeks+unsubscr...@googlegroups.com.
For more options, visit this group at 
http://groups.google.com/group/algogeeks?hl=en.



Re: [algogeeks] Re: exit() vs. _exit()

2011-08-18 Thread Abhishek Sharma
exit() calls clean up codes (flushing the buffer etc) while _exit() does
not..

FYI- pls use google for such questions.

Regards,
Abhishek

On Thu, Aug 18, 2011 at 3:26 PM, Amol Sharma amolsharm...@gmail.com wrote:

 anyone ??

 --


 Amol Sharma
 Third Year Student
 Computer Science and Engineering
 MNNIT Allahabad




 On Thu, Aug 18, 2011 at 10:40 AM, Amol Sharma amolsharm...@gmail.comwrote:

 what is the difference between exit() and _exit

 http://ideone.com/MCzGy

 http://ideone.com/SxbwT

 when i am using exit() hello world is printed but with _exit() nothing
 gets printed though it ran successfully.plz explain why is so happening
 ??


 --


 Amol Sharma
 Third Year Student
 Computer Science and Engineering
 MNNIT Allahabad



  --
 You received this message because you are subscribed to the Google Groups
 Algorithm Geeks group.
 To post to this group, send email to algogeeks@googlegroups.com.
 To unsubscribe from this group, send email to
 algogeeks+unsubscr...@googlegroups.com.
 For more options, visit this group at
 http://groups.google.com/group/algogeeks?hl=en.


-- 
You received this message because you are subscribed to the Google Groups 
Algorithm Geeks group.
To post to this group, send email to algogeeks@googlegroups.com.
To unsubscribe from this group, send email to 
algogeeks+unsubscr...@googlegroups.com.
For more options, visit this group at 
http://groups.google.com/group/algogeeks?hl=en.



Re: [algogeeks] Puzzle

2011-08-18 Thread Abhishek Sharma
M (hint: replace ü and û with their actual meaning.. u 'll understand)

On Fri, Aug 19, 2011 at 4:10 AM, payal gupta gpt.pa...@gmail.com wrote:

 was there anything more specified
 Regards,
 PAYAL GUPTA


 On Fri, Aug 19, 2011 at 3:29 AM, Aditya Virmani 
 virmanisadi...@gmail.comwrote:

 If ü - Married
 û - Not Married and
 M-ü N-û
 N-ü L-û
 L-û M-ü
 Who is married?

 qn was put up in this way, asked in Deloitte 2004.

 --
 You received this message because you are subscribed to the Google Groups
 Algorithm Geeks group.
 To post to this group, send email to algogeeks@googlegroups.com.
 To unsubscribe from this group, send email to
 algogeeks+unsubscr...@googlegroups.com.
 For more options, visit this group at
 http://groups.google.com/group/algogeeks?hl=en.


  --
 You received this message because you are subscribed to the Google Groups
 Algorithm Geeks group.
 To post to this group, send email to algogeeks@googlegroups.com.
 To unsubscribe from this group, send email to
 algogeeks+unsubscr...@googlegroups.com.
 For more options, visit this group at
 http://groups.google.com/group/algogeeks?hl=en.


-- 
You received this message because you are subscribed to the Google Groups 
Algorithm Geeks group.
To post to this group, send email to algogeeks@googlegroups.com.
To unsubscribe from this group, send email to 
algogeeks+unsubscr...@googlegroups.com.
For more options, visit this group at 
http://groups.google.com/group/algogeeks?hl=en.



Re: [algogeeks] Coding..........

2011-07-21 Thread Abhishek Sharma
scan the array from both the ends..i.e use two pointers to scan the array..
left pointer points at the beginning and right one one at the end..
left pointer can be used to find whether an element is even or not until it
reaches end while right pointer can be used to find whthr a no is odd until
it reaches the starting of the array..

for (i=0 until i+ a[].length){

if (*leftptr % 2 == 0)
 A2[i] = *leftptr ;
else
 leftptr++;
if (*rtptr % 2 == 0)
 A2[i+a[].length-1] = *rtptr ;
else
 rtptr--;
}


On Thu, Jul 21, 2011 at 11:32 PM, UMESH KUMAR kumar.umesh...@gmail.comwrote:

 Hi
  Given an array A[], write a function to separate even and odd numbers
 (i.e., put all even numbers first than odd numbers) and stability is also
 maintain for the elements in the Array.

 Eg.
 input:

 A[] = {12, 34, 45, 9, 8, 90, 3}

 output:
 A[] = {12, 34, 8, 90, 45, 9, 3}**




 Thanks

 --
 You received this message because you are subscribed to the Google Groups
 Algorithm Geeks group.
 To post to this group, send email to algogeeks@googlegroups.com.
 To unsubscribe from this group, send email to
 algogeeks+unsubscr...@googlegroups.com.
 For more options, visit this group at
 http://groups.google.com/group/algogeeks?hl=en.


-- 
You received this message because you are subscribed to the Google Groups 
Algorithm Geeks group.
To post to this group, send email to algogeeks@googlegroups.com.
To unsubscribe from this group, send email to 
algogeeks+unsubscr...@googlegroups.com.
For more options, visit this group at 
http://groups.google.com/group/algogeeks?hl=en.



Re: [algogeeks] Coding..........

2011-07-21 Thread Abhishek Sharma
small change in the pseudocode..

for (i=0 until i+ a[].length){

if (*leftptr % 2 == 0)
 A2[i] = *leftptr ;

else if (*rtptr % 2 == 0)
 A2[i+a[].length-1] = *rtptr ;

leftptr++;
rtptr--;

}

-- 
You received this message because you are subscribed to the Google Groups 
Algorithm Geeks group.
To post to this group, send email to algogeeks@googlegroups.com.
To unsubscribe from this group, send email to 
algogeeks+unsubscr...@googlegroups.com.
For more options, visit this group at 
http://groups.google.com/group/algogeeks?hl=en.



Re: [algogeeks] sbtration

2011-07-11 Thread Abhishek Sharma
x-y = add(x, add(~y, 1)) 
here add(~y,1) refers to the Two's complement of y...

On 7/12/11, Anika Jain anika.jai...@gmail.com wrote:
 how to do subtraction of two integers widout using subtractn??

 --
 You received this message because you are subscribed to the Google Groups
 Algorithm Geeks group.
 To post to this group, send email to algogeeks@googlegroups.com.
 To unsubscribe from this group, send email to
 algogeeks+unsubscr...@googlegroups.com.
 For more options, visit this group at
 http://groups.google.com/group/algogeeks?hl=en.



-- 
You received this message because you are subscribed to the Google Groups 
Algorithm Geeks group.
To post to this group, send email to algogeeks@googlegroups.com.
To unsubscribe from this group, send email to 
algogeeks+unsubscr...@googlegroups.com.
For more options, visit this group at 
http://groups.google.com/group/algogeeks?hl=en.



Re: [algogeeks] os

2011-06-23 Thread Abhishek Sharma
@rahul: buddy, u can ignore the mail if u don't want to answer (no offense).
Lets not discourage someone from asking questions...

On Tue, Jun 21, 2011 at 11:23 PM, rahul rahulr...@gmail.com wrote:

 If u want us to solve the GATE paper, please attach the paper, we will post
 the solution.

 regards.


 On Tue, Jun 21, 2011 at 11:21 PM, Akshata Sharma 
 akshatasharm...@gmail.com wrote:

 The atomic fetch-and-set x, y instruction unconditionally sets the memory
 location x to 1 and fetches the old value of x n y without allowing any
 intervening access to the memory location x. consider the following
 implementation of P and V functions on a binary semaphore S.

 void P (binary_semaphore *s)
   unsigned y;
   unsigned * = (s—value);
   do
 fetch—and—set x, y;
   while (y)

 void V (binary_semaphore *s)
S—value = 0;
 Which one of the following is true?
 (A) The implementation may not work if context switching is disabled in P
 (B) Instead of using fetch-and —set, a pair of normal load/store can be
 used
 (C) The implementation of V is wrong
 (D) The code does not implement a binary semaphore

 --
 You received this message because you are subscribed to the Google Groups
 Algorithm Geeks group.
 To post to this group, send email to algogeeks@googlegroups.com.
 To unsubscribe from this group, send email to
 algogeeks+unsubscr...@googlegroups.com.
 For more options, visit this group at
 http://groups.google.com/group/algogeeks?hl=en.


  --
 You received this message because you are subscribed to the Google Groups
 Algorithm Geeks group.
 To post to this group, send email to algogeeks@googlegroups.com.
 To unsubscribe from this group, send email to
 algogeeks+unsubscr...@googlegroups.com.
 For more options, visit this group at
 http://groups.google.com/group/algogeeks?hl=en.


-- 
You received this message because you are subscribed to the Google Groups 
Algorithm Geeks group.
To post to this group, send email to algogeeks@googlegroups.com.
To unsubscribe from this group, send email to 
algogeeks+unsubscr...@googlegroups.com.
For more options, visit this group at 
http://groups.google.com/group/algogeeks?hl=en.



Re: [algogeeks] Online Jobs for Students

2011-04-16 Thread Abhishek Sharma
Our all candidates of batch Sept 10 - Jan 11 got placed in top MNC companies
with very lucrative salary like earlier batches. Most of the candidates had
multiple job offers with min Rs. 15000/-  in hand per month salary. Jan 11 -
May 11 batch placement is running and almost 50% students has already got
placed with average salary of Rs. 17,500/- per month.  After a huge
recommendations and orders from many MNC and Pvt. Ltd companies, we are
going to start a new batch April 2011 of iphone/Android Application and game
development namely,

*1. Flab Java -* Android Application and 2D/ 3D Game Development course
for six months. You will be trained completely on Advance Android
application development, Device driver programming and 2D/3D Game
development.

*2. Flab Macintosh -* iphone/ipad Application and 2D/3D Game Development
course for six months. You will be trained completely on Advance iphone/ipad
application development, Device driver programming for Macintosh and 2D/3D
Game development.

*Placement at FresherLab*

The quality of training at FresherLab has reflected in its student’s
extraordinary performance in Placement session. All trained students have
been placed in top MNC companies within fifteen days of start of placement
session. Around 80% students had multiples job offer from different MNC
companies. FresherLab provides high quality placement services to its
trained candidates.

The average salary offered to FresherLab's trained candidate of earlier
batches was 2.2 Lac per Annum with INR 16,000 per month in hand. The list of
the trained candidates from FresherLab who was offered the placement in MNC
and Pvt. Limited companies in the batch of Sept.10- Jan.11 can be viewed by
*clicking this link
*http://www.fresherslab.com/email/link.php?M=798426N=167L=4F=H

Based on the quality training and getting deserving candidates from
FresherLab, companies have tied up with FresherLab for a long term placement
and need trained professional from us. So again, FresherLab is back with the
training as per company's order. There are huge requirements of iphone/ipad
and Android application developer in  India and abroad.

*Please click here to download complete placement detail and List of
companies who recruit our students.
http://www.fresherslab.com/email/link.php?M=798426N=167L=5F=H
*

*No one can promise in written document, but we will give you written
agreement for 100% job guarantee:*
We are providing 100% job guarantee with this course. In case you couldn't
get placed in four  months after course completion, we will refund your
money back. We will give the agreement in written.

*Course syllabus*

We are attaching the complete course syllabus with this email. In brief, you
will be trained like this.

 1. First Two Months: Complete application development from scratches with
three live projects.

 2. Third Month : Live project based on training done in first two months.

 3. Forth Month: 2D Game Development with one live project .

 4. Fifth Month: 3D Game Development with one live project.

 5. Sixth Month : Game development Live project.

 This Six Months course has been designed such a way that you will get
experience equivalent to 1.5 years in the industries with minimum 5 live
projects.

*You can download the detailed course syllabus from the following links.*

 1. iphone/ipad application and game development : *click here to
download course
syllabushttp://www.fresherslab.com/email/link.php?M=798426N=167L=2F=H
 *

 2. Android application and game development : *click here to download
course http://www.fresherslab.com/email/link.php?M=798426N=167L=3F=H **
syllabus http://www.fresherslab.com/email/link.php?M=798426N=167L=3F=H*

*Certification *

We are offering following certifications with this course.

1. Expert Rating certified Android Application Developer

2. Expert Rating certified iphone/ipad Application Developer

3. Fresher Lab Certified Android Application Developer

4. Fresher Lab Certified iphone/ipad Application Developer

5. Fresher Lab Certified 2D Game Developer

6. Fresher Lab Certified 3D Game Developer

This course has been specially designed to qualify above certifications
which will be giving value to your future career.

*Fees
*
The course fee for this course is Rs. 65000/- .
We are offering installment option as well. Following is the installment
option.
 1. Lumpsum: Rs. 65000/-
 2. Two Installments: Rs. 35,000/- each
*Join us
*
 We are making a batch of only 16 candidates in iphone/ipad and 9 candidates
in Android batch, As we have got order from many MNC and Pvt. Limited
companies to provide Android and iphone/ipad trained Freshers, That is the
reason, we are starting this batch with 100% job guarantee. The enrollment
for the batch has been started.
*
 *
For any enquiry, you can call us at following numbers:
*Mobile No. : +91- 9241051869, 9241581071, 9241409209, 9241867775
 *
If you are Interested, you can enroll on or *before 20th April 2011.
 *

Re: [algogeeks] Re: 28march

2011-04-06 Thread Abhishek Sharma
@sourabh: could u please elaborate how u came to that conclusion.
Dave's logic seems to be right..

On Thu, Mar 31, 2011 at 11:00 AM, sourabh jakhar sourabhjak...@gmail.comwrote:

 answer is 6 races



 On Mon, Mar 28, 2011 at 11:53 PM, Dave dave_and_da...@juno.com wrote:

 7 races.

 For the first five races, divide the horses into groups of five and
 record the win, place, and show finishers of each race.

 For the sixth race, run the winners of the first five races.

 Now, only six horses remain in contention for the fastest three:
   The winner of the sixth race and the place and show horses of his
 first race,
   The place horse in the sixth race and the place horse in his first
 race.
   The show horse in the sixth race.
   Three of these horses are known to be faster than all other horses.

 The winner of the sixth race is known to be the fastest horse. Run the
 other five contenders in race 7 and choose the fastest two.

 Dave

 On Mar 28, 2:54 am, Lavesh Rawat lavesh.ra...@gmail.com wrote:
  *Horse Race Problem Solution*
  *
  *Ok, so there are 25 horses and the race track only allows 5 horses to
 race
  at a given time. Given that there is no stop watch available your task
 is to
  determine the fastest 3 horses. Assume that each horses speed is
 constant in
  different races, what is the minimum number of races to determine the
  fastest 3?
 
  Update Your Answers at : Click
  Here
 http://dailybrainteaser.blogspot.com/2011/03/28march.html?lavesh=lavesh
 
  Solution:
  Will be updated after 1 day
 
  --
 
  Never explain yourself. Your friends don’t need it
 and
  your enemies won’t believe it .

 --
 You received this message because you are subscribed to the Google Groups
 Algorithm Geeks group.
 To post to this group, send email to algogeeks@googlegroups.com.
 To unsubscribe from this group, send email to
 algogeeks+unsubscr...@googlegroups.com.
 For more options, visit this group at
 http://groups.google.com/group/algogeeks?hl=en.




 --
 SOURABH JAKHAR,(CSE)(3 year)
 ROOM NO 167 ,
 TILAK,HOSTEL
 'MNNIT ALLAHABAD

 The Law of Win says, Let's not do it your way or my way; let's do it the
 best way.

  --
 You received this message because you are subscribed to the Google Groups
 Algorithm Geeks group.
 To post to this group, send email to algogeeks@googlegroups.com.
 To unsubscribe from this group, send email to
 algogeeks+unsubscr...@googlegroups.com.
 For more options, visit this group at
 http://groups.google.com/group/algogeeks?hl=en.


-- 
You received this message because you are subscribed to the Google Groups 
Algorithm Geeks group.
To post to this group, send email to algogeeks@googlegroups.com.
To unsubscribe from this group, send email to 
algogeeks+unsubscr...@googlegroups.com.
For more options, visit this group at 
http://groups.google.com/group/algogeeks?hl=en.



Re: [algogeeks] Are U a Student Must Read this Frwd This Please If u Like We R student like You TOO

2011-04-06 Thread Abhishek Sharma
@Carl: the one at the bottom works..

On Fri, Apr 1, 2011 at 1:17 AM, hary rathor harry.rat...@gmail.com wrote:

 everybody want to be mark.

 --
 You received this message because you are subscribed to the Google Groups
 Algorithm Geeks group.
 To post to this group, send email to algogeeks@googlegroups.com.
 To unsubscribe from this group, send email to
 algogeeks+unsubscr...@googlegroups.com.
 For more options, visit this group at
 http://groups.google.com/group/algogeeks?hl=en.


-- 
You received this message because you are subscribed to the Google Groups 
Algorithm Geeks group.
To post to this group, send email to algogeeks@googlegroups.com.
To unsubscribe from this group, send email to 
algogeeks+unsubscr...@googlegroups.com.
For more options, visit this group at 
http://groups.google.com/group/algogeeks?hl=en.



Re: [algogeeks] interview

2011-03-27 Thread Abhishek Sharma
*@Shady:* buddy lets not discourage som1 interested in learning :) this is
the purpose of this group..
*
@harry:* as some1 has said: there is no shortcut to success.. first go
through any good Datastructures/algorithms book.. try to code those
algorithms on ur own..once u feel u r comfortable with the basics u can go
through the previous posts of this group and implement the algos for more
practice..


On Mon, Mar 28, 2011 at 12:22 AM, shady sinv...@gmail.com wrote:

 what kind of question is this ?
 the quality of this group is degrading day by day :(


 On Sun, Mar 27, 2011 at 10:44 PM, hary rathor harry.rat...@gmail.comwrote:

 hello friends
 pls suggest me that which algorithm problem i should implement  that are
 ask  in most in interviews

 --
 You received this message because you are subscribed to the Google Groups
 Algorithm Geeks group.
 To post to this group, send email to algogeeks@googlegroups.com.
 To unsubscribe from this group, send email to
 algogeeks+unsubscr...@googlegroups.com.
 For more options, visit this group at
 http://groups.google.com/group/algogeeks?hl=en.


  --
 You received this message because you are subscribed to the Google Groups
 Algorithm Geeks group.
 To post to this group, send email to algogeeks@googlegroups.com.
 To unsubscribe from this group, send email to
 algogeeks+unsubscr...@googlegroups.com.
 For more options, visit this group at
 http://groups.google.com/group/algogeeks?hl=en.


-- 
You received this message because you are subscribed to the Google Groups 
Algorithm Geeks group.
To post to this group, send email to algogeeks@googlegroups.com.
To unsubscribe from this group, send email to 
algogeeks+unsubscr...@googlegroups.com.
For more options, visit this group at 
http://groups.google.com/group/algogeeks?hl=en.



Re: [algogeeks] Print Hello infinite..................

2011-03-15 Thread Abhishek Sharma
thanks a ton for the info... we didnt know about that :P...
btw for ur info.. we are not supposed to use loops as well..

On Fri, Mar 11, 2011 at 7:10 PM, Abhishek Mallick 
abhishek.mallick2...@gmail.com wrote:

 #include stdio.h
 int main()
 {
 while(printf(Hello));
 return 0;
 }

 On Thu, Mar 10, 2011 at 11:58 AM, Nishant Agarwal 
 nishant.agarwa...@gmail.com wrote:

 #includestdio.h
 void print1();
 void print2()
 {
 printf(Hello\n);
 print1();
 }
 void print1()
 {
 printf(Hello\n);
 print2();
 }
 int main()
 {
 print1();

 }


 On Thu, Mar 10, 2011 at 11:47 AM, nidhi jain 
 nidhi.jain311...@gmail.comwrote:



 @abhishek:isn't it recursion?

  --
 You received this message because you are subscribed to the Google Groups
 Algorithm Geeks group.
 To post to this group, send email to algogeeks@googlegroups.com.
 To unsubscribe from this group, send email to
 algogeeks+unsubscr...@googlegroups.com.
 For more options, visit this group at
 http://groups.google.com/group/algogeeks?hl=en.





  --
 You received this message because you are subscribed to the Google Groups
 Algorithm Geeks group.
 To post to this group, send email to algogeeks@googlegroups.com.
 To unsubscribe from this group, send email to
 algogeeks+unsubscr...@googlegroups.com.
 For more options, visit this group at
 http://groups.google.com/group/algogeeks?hl=en.


  --
 You received this message because you are subscribed to the Google Groups
 Algorithm Geeks group.
 To post to this group, send email to algogeeks@googlegroups.com.
 To unsubscribe from this group, send email to
 algogeeks+unsubscr...@googlegroups.com.
 For more options, visit this group at
 http://groups.google.com/group/algogeeks?hl=en.


-- 
You received this message because you are subscribed to the Google Groups 
Algorithm Geeks group.
To post to this group, send email to algogeeks@googlegroups.com.
To unsubscribe from this group, send email to 
algogeeks+unsubscr...@googlegroups.com.
For more options, visit this group at 
http://groups.google.com/group/algogeeks?hl=en.



Re: [algogeeks] Print Hello infinite..................

2011-03-15 Thread Abhishek Sharma
@nidhi: yup.. u r rite.. sorry..my bad...

On Thu, Mar 10, 2011 at 11:58 AM, Nishant Agarwal 
nishant.agarwa...@gmail.com wrote:

 #includestdio.h
 void print1();
 void print2()
 {
 printf(Hello\n);
 print1();
 }
 void print1()
 {
 printf(Hello\n);
 print2();
 }
 int main()
 {
 print1();

 }


 On Thu, Mar 10, 2011 at 11:47 AM, nidhi jain 
 nidhi.jain311...@gmail.comwrote:



 @abhishek:isn't it recursion?

  --
 You received this message because you are subscribed to the Google Groups
 Algorithm Geeks group.
 To post to this group, send email to algogeeks@googlegroups.com.
 To unsubscribe from this group, send email to
 algogeeks+unsubscr...@googlegroups.com.
 For more options, visit this group at
 http://groups.google.com/group/algogeeks?hl=en.





  --
 You received this message because you are subscribed to the Google Groups
 Algorithm Geeks group.
 To post to this group, send email to algogeeks@googlegroups.com.
 To unsubscribe from this group, send email to
 algogeeks+unsubscr...@googlegroups.com.
 For more options, visit this group at
 http://groups.google.com/group/algogeeks?hl=en.


-- 
You received this message because you are subscribed to the Google Groups 
Algorithm Geeks group.
To post to this group, send email to algogeeks@googlegroups.com.
To unsubscribe from this group, send email to 
algogeeks+unsubscr...@googlegroups.com.
For more options, visit this group at 
http://groups.google.com/group/algogeeks?hl=en.



Re: [algogeeks] Print Hello infinite..................

2011-03-09 Thread Abhishek Sharma
#include headers

int main(){

printf(Hello);
main();
}


On Mon, Mar 7, 2011 at 7:38 AM, Terence technic@gmail.com wrote:

  system(yes Hello);
 (on Linux)


 On 2011-3-7 2:09, sudheer kumar wrote:

 use GOTO

 On Sun, Mar 6, 2011 at 10:49 PM, UMESH KUMAR kumar.umesh...@gmail.comwrote:

 How to print Hello Infinite times without using
 Loop and Recursion function  ?
 --
  You received this message because you are subscribed to the Google Groups
 Algorithm Geeks group.
 To post to this group, send email to algogeeks@googlegroups.com.
 To unsubscribe from this group, send email to
 algogeeks+unsubscr...@googlegroups.com.
 For more options, visit this group at
 http://groups.google.com/group/algogeeks?hl=en.




 --
 Thanks and Regards
 chigullapallysudh...@gmail.com
 Sudheer

 --
 You received this message because you are subscribed to the Google Groups
 Algorithm Geeks group.
 To post to this group, send email to algogeeks@googlegroups.com.
 To unsubscribe from this group, send email to
 algogeeks+unsubscr...@googlegroups.com.
 For more options, visit this group at
 http://groups.google.com/group/algogeeks?hl=en.

  --
 You received this message because you are subscribed to the Google Groups
 Algorithm Geeks group.
 To post to this group, send email to algogeeks@googlegroups.com.
 To unsubscribe from this group, send email to
 algogeeks+unsubscr...@googlegroups.com.
 For more options, visit this group at
 http://groups.google.com/group/algogeeks?hl=en.


-- 
You received this message because you are subscribed to the Google Groups 
Algorithm Geeks group.
To post to this group, send email to algogeeks@googlegroups.com.
To unsubscribe from this group, send email to 
algogeeks+unsubscr...@googlegroups.com.
For more options, visit this group at 
http://groups.google.com/group/algogeeks?hl=en.



Re: [algogeeks] Interview questions for multithreading in Java

2011-03-08 Thread Abhishek Sharma
u can go through the following questions...not sure about the answers...:

1) What are the two types of multitasking?

Ans : 1.process-based

2.Thread-based

2) What are the two ways to create the thread?

Ans : 1.by implementing Runnable

2.by extending Thread

3) What is the signature of the constructor of a thread class?

Ans : Thread(Runnable threadob,String threadName)

4) What are all the methods available in the Runnable Interface?

Ans : run()

5) What is the data type for the method isAlive() and this method is

available in which class?

Ans : boolean, Thread

6) What are all the methods available in the Thread class?

Ans : 1.isAlive()

2.join()

3.resume()

4.suspend()

5.stop()

6.start()

7.sleep()

8.destroy()

7) What are all the methods used for Inter Thread communication and what is
the class in which these methods are defined?

Ans :1. wait(),notify()  notifyall()

2. Object class

8) What is the mechanisam defind by java for the Resources to be used by
only one Thread at a time?

Ans : Synchronisation

9) What is the procedure to own the moniter by many threads?

Ans : not possible

10) What is the unit for 1000 in the below statement?

ob.sleep(1000)

Ans : long milliseconds

11) What is the data type for the parameter of the sleep() method?

Ans : long

12) What are all the values for the following level?

max-priority

min-priority

normal-priority

Ans : 10,1,5

13) What is the method available for setting the priority?

Ans : setPriority()

14) What is the default thread at the time of starting the program?

Ans : main thread

15) The word synchronized can be used with only a method.

True/ False

Ans : False

16) Which priority Thread can prompt the lower primary Thread?

Ans : Higher Priority

17) How many threads at a time can access a monitor?

Ans : one

18) What are all the four states associated in the thread?

Ans : 1. new 2. runnable 3. blocked 4. dead

19) The suspend()method is used to teriminate a thread?

True /False

Ans : False

20) The run() method should necessary exists in clases created as subclass
of thread?

True /False

Ans : True

21) When two threads are waiting on each other and can't proceed the
programe is said to be in a deadlock?

True/False

Ans : True

22) Which method waits for the thread to die ?

Ans : join() method

23) Which of the following is true?

1) wait(),notify(),notifyall() are defined as final  can be called only
from with in a synchronized method

2) Among wait(),notify(),notifyall() the wait() method only throws
IOException

3) wait(),notify(),notifyall()  sleep() are methods of object class

1
2
3
1 amp; 2
1,2  3
Ans : D
24) Garbage collector thread belongs to which priority?

Ans : low-priority

25) What is meant by timeslicing or time sharing?

Ans : Timeslicing is the method of allocating CPU time to individual threads
in a priority schedule.

26) What is meant by daemon thread? In java runtime, what is it's role?

Ans : Daemon thread is a low priority thread which runs intermittently in
the background doing the garbage collection operation for the java runtime
system

On Tue, Mar 8, 2011 at 9:16 PM, Abhishek Sharma jkabhishe...@gmail.comwrote:

 what do u mean by multi-threading :P


 On Tue, Mar 8, 2011 at 7:37 PM, vaibhav agrawal agrvaib...@gmail.comwrote:

 Hi,

 What interview questions one would expect for multi-threading?

 Thanks,
 Vaibhav

 --
 You received this message because you are subscribed to the Google Groups
 Algorithm Geeks group.
 To post to this group, send email to algogeeks@googlegroups.com.
 To unsubscribe from this group, send email to
 algogeeks+unsubscr...@googlegroups.com.
 For more options, visit this group at
 http://groups.google.com/group/algogeeks?hl=en.




-- 
You received this message because you are subscribed to the Google Groups 
Algorithm Geeks group.
To post to this group, send email to algogeeks@googlegroups.com.
To unsubscribe from this group, send email to 
algogeeks+unsubscr...@googlegroups.com.
For more options, visit this group at 
http://groups.google.com/group/algogeeks?hl=en.



Re: [algogeeks] [brain teaser ] 1march

2011-03-01 Thread Abhishek Sharma
beetle :P

On Tue, Mar 1, 2011 at 2:01 PM, Ankur pratik ankurocks...@gmail.com wrote:

 beetle

  --
 You received this message because you are subscribed to the Google Groups
 Algorithm Geeks group.
 To post to this group, send email to algogeeks@googlegroups.com.
 To unsubscribe from this group, send email to
 algogeeks+unsubscr...@googlegroups.com.
 For more options, visit this group at
 http://groups.google.com/group/algogeeks?hl=en.


-- 
You received this message because you are subscribed to the Google Groups 
Algorithm Geeks group.
To post to this group, send email to algogeeks@googlegroups.com.
To unsubscribe from this group, send email to 
algogeeks+unsubscr...@googlegroups.com.
For more options, visit this group at 
http://groups.google.com/group/algogeeks?hl=en.



Re: [algogeeks] Maths

2011-02-22 Thread Abhishek Sharma
This is simple..

Find the values for f(n) for n=1,2,3,4,... n-1 which are 0, 1, 2, 3, ... n-2
respectively. (Solve the equation for n=2,3, etc to get the values).

From the pattern you can easily find out that f(n+1)= n.

On Wed, Feb 16, 2011 at 9:15 PM, Vikas Kumar dev.vika...@gmail.com wrote:

 f(n)=n-1.


 On Wed, Feb 16, 2011 at 7:39 PM, Akshata Sharma akshatasharm...@gmail.com
  wrote:

 please help..

 if f(n+1) = max{ f(k) + f(n-k+1) + 1} for 1 = k = n; f(1)  = 0.
 Find f(n+1) in terms of n.
 Eg: f(4) = ? n = 3; 1= k = 3; f(4) = max{f(1) + f(3) + 1, f(2) +
 f(2)+1, f(3) + f(1) +1}

 --
 You received this message because you are subscribed to the Google Groups
 Algorithm Geeks group.
 To post to this group, send email to algogeeks@googlegroups.com.
 To unsubscribe from this group, send email to
 algogeeks+unsubscr...@googlegroups.com.
 For more options, visit this group at
 http://groups.google.com/group/algogeeks?hl=en.


  --
 You received this message because you are subscribed to the Google Groups
 Algorithm Geeks group.
 To post to this group, send email to algogeeks@googlegroups.com.
 To unsubscribe from this group, send email to
 algogeeks+unsubscr...@googlegroups.com.
 For more options, visit this group at
 http://groups.google.com/group/algogeeks?hl=en.


-- 
You received this message because you are subscribed to the Google Groups 
Algorithm Geeks group.
To post to this group, send email to algogeeks@googlegroups.com.
To unsubscribe from this group, send email to 
algogeeks+unsubscr...@googlegroups.com.
For more options, visit this group at 
http://groups.google.com/group/algogeeks?hl=en.



Re: [algogeeks] can i know the best way to learn programming??

2011-02-02 Thread Abhishek Sharma
dude... Practice is the only solution to ur problem...

U can try any one of the following: topcoder, spoj, (already mentioned in
the thread) codechef etc...

U can also refer to previous posts in this group and try to code those
problems on your own and then refer to the solutions to check where you can
improve..U can always come back to us in case of any difficulty..

Happy Coding :)

-- 
You received this message because you are subscribed to the Google Groups 
Algorithm Geeks group.
To post to this group, send email to algogeeks@googlegroups.com.
To unsubscribe from this group, send email to 
algogeeks+unsubscr...@googlegroups.com.
For more options, visit this group at 
http://groups.google.com/group/algogeeks?hl=en.



Re: [algogeeks] Sites for Interview Questions

2011-01-21 Thread Abhishek Sharma
Programming Interviews Exposed is a good one..

On Tue, Jan 18, 2011 at 9:27 PM, Yellow Sapphire pukhraj7...@gmail.comwrote:

 Hi,

 Can someone suggest good books/websites/blogs for interview related
 questions.


 thanks--
 YS

 --
 You received this message because you are subscribed to the Google Groups
 Algorithm Geeks group.
 To post to this group, send email to algogeeks@googlegroups.com.
 To unsubscribe from this group, send email to
 algogeeks+unsubscr...@googlegroups.comalgogeeks%2bunsubscr...@googlegroups.com
 .
 For more options, visit this group at
 http://groups.google.com/group/algogeeks?hl=en.


-- 
You received this message because you are subscribed to the Google Groups 
Algorithm Geeks group.
To post to this group, send email to algogeeks@googlegroups.com.
To unsubscribe from this group, send email to 
algogeeks+unsubscr...@googlegroups.com.
For more options, visit this group at 
http://groups.google.com/group/algogeeks?hl=en.



[algogeeks] Algorithm

2010-07-05 Thread Abhishek Sharma
You are given a set of phrases (let us call them keywords), each of
1-4 words. Group the keywords into clusters (groups of keywords) by
picking keywords which are similar to each other.

There are multiple ways to cluster(group) them
1) using single keyword as the cluster name. Eg cluster name gifts -
this cluster should have all keywords which has gift in it, christmas
- should have all keywords which have christmas in it and so on
2) considering 2 words as the cluster name: gift ideas, gifts,
christmas, craft. All keywords which have the 2 word cluster name
appearing in sequence (phrase matching) in the keyword fall into that
cluster

To pick the cluster name: Pick the most popular 1 word and 2 word
sequence from the given keyword set


Eg input set of keywords:
best dad gifts
best gifts
birthday gift ideas
birthday gifts
boss gift ideas
boyfriend gift ideas
christmas craft ideas
christmas for dad
christmas gifts for dad
christmas gift ideas
christmas gifts
christmas gifts dad
cool gifts
craft gift ideas
craft ideas
craft room ideas
crafts

Sample Clusters:
gifts: best dad gifts, best gifts, birthday gifts, christmas gifts for
dad, christmas gifts, christmas gifts dad, cool gifts
gift: birthday gift ideas, boss gift ideas, boyfriend gift
ideas,christmas gift ideas, craft gift ideas
christmas: christmas craft ideas, christmas for dad, christmas
gifts for dad, christmas gift ideas, christmas gifts, christmas
gifts dad
gift ideas: birthday gift ideas, boyfriend gift ideas, christmas
gift ideas, craft gift ideas

and so on

-- 
You received this message because you are subscribed to the Google Groups 
Algorithm Geeks group.
To post to this group, send email to algoge...@googlegroups.com.
To unsubscribe from this group, send email to 
algogeeks+unsubscr...@googlegroups.com.
For more options, visit this group at 
http://groups.google.com/group/algogeeks?hl=en.



Re: [algogeeks] Project on finding an optimal route given a map.

2010-07-02 Thread Abhishek Sharma
@Anand:
 Let me know your input we can modify it accordingly.
I have already mentioned it in previous posts.. for your sake I ll do it
again..
Input is a a map of a small area..(some college campus)..it can be in the
form of an image, osm format (www.openstreetmap.org) or in the kml format (
maps.google.com) for that particular region...
So creating distance matrix is the problem.. I do not want to do it
manually(ie finding the distance of each node/dept from other nodes manually
and then updating the distance matrix).. so i was thinking of writing an
algorithm which takes the input and creates the distance matrix... which is
the main challenge in this project.
I request you to join the group..(http://groups.google.co.in/group/*
optiroute* http://groups.google.co.in/group/optiroute ) so that we discuss
about this there..instead of spamming this group..

@Sidhartha: we will face following problems if we use open Route Service:
  1) we ll have to zoom in to the place everytime we
open/refresh the page.
  2) and the main thing is it works only for Europe.
thanks for the help by the way. why dont you also join the group..
http://groups.google.co.in/group/*optiroute*http://groups.google.co.in/group/optiroute

-- 
You received this message because you are subscribed to the Google Groups 
Algorithm Geeks group.
To post to this group, send email to algoge...@googlegroups.com.
To unsubscribe from this group, send email to 
algogeeks+unsubscr...@googlegroups.com.
For more options, visit this group at 
http://groups.google.com/group/algogeeks?hl=en.



Re: [algogeeks] Amazon: sort array

2010-07-02 Thread Abhishek Sharma
@Anand: good one

On Sat, Jul 3, 2010 at 2:02 AM, Anand anandut2...@gmail.com wrote:

 This is an example of bitonic sequence if we reverse the bottom half of the
 array.
 Sequence is called Bitonics if the sequence of number first
 increases(ascending order) and then decrease(descending order).

 1. We need to reverse the bottom half the array to make it bitonic.
 2. Appy Bitonic Merge to get the final sorted array.: Complexity.O(n)

 In the below code, I have implemented sorting n/w to sort any kind of array
 but for bitonic sequence we only bitonic merge function call which take
 O(n).
 Refer section Sorting network from Corman for more details

 http://codepad.org/ZhYEBqMw




 On Fri, Jul 2, 2010 at 11:30 AM, ANKUR BHARDWAJ ankibha...@gmail.comwrote:

 Given an array of n elements and an integer k where kn. Elemnts
 {a[0].a[k] and a[k+1].a[n] are already sorted. Give an
 algorithm to sort in O(n) time and O(1) space.

 --
 You received this message because you are subscribed to the Google Groups
 Algorithm Geeks group.
 To post to this group, send email to algoge...@googlegroups.com.
 To unsubscribe from this group, send email to
 algogeeks+unsubscr...@googlegroups.comalgogeeks%2bunsubscr...@googlegroups.com
 .
 For more options, visit this group at
 http://groups.google.com/group/algogeeks?hl=en.


  --
 You received this message because you are subscribed to the Google Groups
 Algorithm Geeks group.
 To post to this group, send email to algoge...@googlegroups.com.
 To unsubscribe from this group, send email to
 algogeeks+unsubscr...@googlegroups.comalgogeeks%2bunsubscr...@googlegroups.com
 .
 For more options, visit this group at
 http://groups.google.com/group/algogeeks?hl=en.


-- 
You received this message because you are subscribed to the Google Groups 
Algorithm Geeks group.
To post to this group, send email to algoge...@googlegroups.com.
To unsubscribe from this group, send email to 
algogeeks+unsubscr...@googlegroups.com.
For more options, visit this group at 
http://groups.google.com/group/algogeeks?hl=en.



Re: [algogeeks] Amazon: sort array

2010-07-02 Thread Abhishek Sharma
I think its similar to the merge operation which is used in merge sort...

correct me if I am wrong..

Regards,
Abhishek

On Sat, Jul 3, 2010 at 12:00 AM, ANKUR BHARDWAJ ankibha...@gmail.comwrote:

 Given an array of n elements and an integer k where kn. Elemnts
 {a[0].a[k] and a[k+1].a[n] are already sorted. Give an
 algorithm to sort in O(n) time and O(1) space.

 --
 You received this message because you are subscribed to the Google Groups
 Algorithm Geeks group.
 To post to this group, send email to algoge...@googlegroups.com.
 To unsubscribe from this group, send email to
 algogeeks+unsubscr...@googlegroups.comalgogeeks%2bunsubscr...@googlegroups.com
 .
 For more options, visit this group at
 http://groups.google.com/group/algogeeks?hl=en.



-- 
You received this message because you are subscribed to the Google Groups 
Algorithm Geeks group.
To post to this group, send email to algoge...@googlegroups.com.
To unsubscribe from this group, send email to 
algogeeks+unsubscr...@googlegroups.com.
For more options, visit this group at 
http://groups.google.com/group/algogeeks?hl=en.



Re: [algogeeks] Project on finding an optimal route given a map.

2010-06-28 Thread Abhishek Sharma
@senthil: thanks for the interest.. I did that purposely..(just wanted to
see if any1 is interested or not).. here are the details...

I have a map..of a small area (say a college campus).. in OSM(openstreetmap)
format or it could also be in kml (google map) format..
Now the application is supposed to take two points on the map as input and
display the optimal/shortest route between them..
For ex: consider any college campus.. user enters ITY dept as the source and
CSE Dept as the destination.. then our application is supposed to display
the shortest/optimal path.
We can also take into account the modee of transport..
Right now.. I am going through the OSM maps.. my idea is to classify the map
into nodes, ways etc.. then applying the algorithm to find the shortest
path..
The problem which i am facing is to classify the map into nodes, ways,
finding out the distance between each node..
some of you might be having a better idea in implementing this... I request
you to share it here..

Guys we have discussed lot of algos here.. but this requires the application
of whatever we have learned...so please come forward and lets implement
this...
hoping for a positive response...

Regards,
Abhishek

-- 
You received this message because you are subscribed to the Google Groups 
Algorithm Geeks group.
To post to this group, send email to algoge...@googlegroups.com.
To unsubscribe from this group, send email to 
algogeeks+unsubscr...@googlegroups.com.
For more options, visit this group at 
http://groups.google.com/group/algogeeks?hl=en.



Re: [algogeeks] Can you solve this?

2010-05-30 Thread Abhishek Sharma
@sharad: if you find the subarrays of equal sum then the number of players
might differ in the team... also can you tell me how will you do
that..according to me time cmoplexity will be higher..

According to me:
sort the palyers based on skill points (O(nlogn) --mergesort) then assign
the players one by one to each team (O(n))

Ex: Consider 10 players to be assigned to two teams.
   skill points: 12, 12, 7, 8, 15, 19, 11, 14, 5, 10.

Ans:
after sorting: 5, 7, 8, 10, 11, 12, 12, 14, 15, 19.
Team1: 5, 8, 12, 14, 19
Team2: 7, 11,12,15.

This is not exactly even but i think is the closest approach.
correct me if I am wrong..

Regards,
Abhishek

On Sun, May 30, 2010 at 8:21 PM, sharad kumar aryansmit3...@gmail.comwrote:

 sort the players based on skill point and get the subarray of equal
 sum..


 On Sun, May 30, 2010 at 6:58 PM, Veer Sharma thisisv...@rediffmail.comwrote:

 Hi Friends,

 This is my first post to this forum. A Hi to all of you and here is
 my first problem...

 Giiven int n, the total number of players and their skill-point.
 Distribute the players on 2 evenly balanced teams.

 Lets see who gives the best solution (least space complexity / least
 time complexity or both...)

 --
 You received this message because you are subscribed to the Google Groups
 Algorithm Geeks group.
 To post to this group, send email to algoge...@googlegroups.com.
 To unsubscribe from this group, send email to
 algogeeks+unsubscr...@googlegroups.comalgogeeks%2bunsubscr...@googlegroups.com
 .
 For more options, visit this group at
 http://groups.google.com/group/algogeeks?hl=en.




 --
 yezhu malai vaasa venkataramana Govinda Govinda

  --
 You received this message because you are subscribed to the Google Groups
 Algorithm Geeks group.
 To post to this group, send email to algoge...@googlegroups.com.
 To unsubscribe from this group, send email to
 algogeeks+unsubscr...@googlegroups.comalgogeeks%2bunsubscr...@googlegroups.com
 .
 For more options, visit this group at
 http://groups.google.com/group/algogeeks?hl=en.


-- 
You received this message because you are subscribed to the Google Groups 
Algorithm Geeks group.
To post to this group, send email to algoge...@googlegroups.com.
To unsubscribe from this group, send email to 
algogeeks+unsubscr...@googlegroups.com.
For more options, visit this group at 
http://groups.google.com/group/algogeeks?hl=en.



Re: [algogeeks] Can you solve this?

2010-05-30 Thread Abhishek Sharma
Correction: Team2:7, 10, 11, 12, 15

On Sun, May 30, 2010 at 11:45 PM, Abhishek Sharma jkabhishe...@gmail.comwrote:

 @sharad: if you find the subarrays of equal sum then the number of players
 might differ in the team... also can you tell me how will you do
 that..according to me time cmoplexity will be higher..

 According to me:
 sort the palyers based on skill points (O(nlogn) --mergesort) then assign
 the players one by one to each team (O(n))

 Ex: Consider 10 players to be assigned to two teams.
skill points: 12, 12, 7, 8, 15, 19, 11, 14, 5, 10.

 Ans:
 after sorting: 5, 7, 8, 10, 11, 12, 12, 14, 15, 19.
 Team1: 5, 8, 12, 14, 19
 Team2: 7, 11,12,15.

 This is not exactly even but i think is the closest approach.
 correct me if I am wrong..

 Regards,
 Abhishek


 On Sun, May 30, 2010 at 8:21 PM, sharad kumar aryansmit3...@gmail.comwrote:

 sort the players based on skill point and get the subarray of equal
 sum..


 On Sun, May 30, 2010 at 6:58 PM, Veer Sharma 
 thisisv...@rediffmail.comwrote:

 Hi Friends,

 This is my first post to this forum. A Hi to all of you and here is
 my first problem...

 Giiven int n, the total number of players and their skill-point.
 Distribute the players on 2 evenly balanced teams.

 Lets see who gives the best solution (least space complexity / least
 time complexity or both...)

 --
 You received this message because you are subscribed to the Google Groups
 Algorithm Geeks group.
 To post to this group, send email to algoge...@googlegroups.com.
 To unsubscribe from this group, send email to
 algogeeks+unsubscr...@googlegroups.comalgogeeks%2bunsubscr...@googlegroups.com
 .
 For more options, visit this group at
 http://groups.google.com/group/algogeeks?hl=en.




 --
 yezhu malai vaasa venkataramana Govinda Govinda

  --
 You received this message because you are subscribed to the Google Groups
 Algorithm Geeks group.
 To post to this group, send email to algoge...@googlegroups.com.
 To unsubscribe from this group, send email to
 algogeeks+unsubscr...@googlegroups.comalgogeeks%2bunsubscr...@googlegroups.com
 .
 For more options, visit this group at
 http://groups.google.com/group/algogeeks?hl=en.




-- 
You received this message because you are subscribed to the Google Groups 
Algorithm Geeks group.
To post to this group, send email to algoge...@googlegroups.com.
To unsubscribe from this group, send email to 
algogeeks+unsubscr...@googlegroups.com.
For more options, visit this group at 
http://groups.google.com/group/algogeeks?hl=en.